You are on page 1of 69

Solutions Manual for

AUCTION THEORY
Alexey Kushnir and Jun Xiao

August 2009

Contents
2 Private Value Auctions: A First Look . . . . . . . . . . . . . . . . . . . . . 2
3 The Revenue Equivalence Principle . . . . . . . . . . . . . . . . . . . . . . . 8
4 Quali…cations and Extensions . . . . . . . . . . . . . . . . . . . . . . . . . . 11
5 Mechanism Design . . . . . . . . . . . . . . . . . . . . . . . . . . . . . . . . 17
6 Auctions with Interdependent Values . . . . . . . . . . . . . . . . . . . . . . 25
8 Asymmetries and Other Complications . . . . . . . . . . . . . . . . . . . . . 34
9 E¢ ciency and the English Auction . . . . . . . . . . . . . . . . . . . . . . . 40
10 Mechanism Design with Interdependent Values . . . . . . . . . . . . . . . . 43
11 Bidding Rings . . . . . . . . . . . . . . . . . . . . . . . . . . . . . . . . . . . 48
13 Equilibrium and E¢ ciency with Private Values . . . . . . . . . . . . . . . . 52
15 Sequential Sales . . . . . . . . . . . . . . . . . . . . . . . . . . . . . . . . . . 55
16 Nonidential Objects . . . . . . . . . . . . . . . . . . . . . . . . . . . . . . . 60
17 Packages and Positions . . . . . . . . . . . . . . . . . . . . . . . . . . . . . . 62

V. Krishna, Auction Theory (2nd. Ed.), Elsevier, 2009.

1
2 Private Value Auctions: A First Look
Problem 2.1 (Power distribution) Suppose there are two bidders with private values
that are distributed independently according to the distribution F (x) = xa over [0; 1]
where a > 0: Find symmetric equilibrium bidding strategies in a …rst-price auction.

Solution. Since N = 2, G(x) = F (x) = xa : Thus, using the formula on page 16 of


the text, Z x Z x a
I G (y) y a
(x) = x dy = x a
dy = x
0 G (x) 0 x 1+a

Problem 2.2 (Pareto distribution) Suppose there are two bidders with private values
that are distributed independently according to a Pareto distribution F (x) = 1
(x + 1) 2 over [0; 1). Find symmetric equilibrium bidding strategies in a …rst-price
auction. Show by direct computation that the expected revenues in a …rst- and second-
price auction are the same.

Solution. Again, since N = 2, G (x) = F (x) = 1 (x + 1) 2 . Thus,


Z x
I G (y)
(x) = x dy
0 G (x)
Z x
1 (y + 1) 2
= x dy
0 1 (x + 1) 2
x
=
x+2
In the …rst-price auction, the expected revenue of the seller is

E RI = 2E mI (x)
I
= 2E G (x) (x)
Z 1
2 x 3
= 2 1 (x + 1) 2 (x + 1) dx
0 x+2
= 1=3

Let Y2 be the second highest value, and its density is f2 (y) = 2 (1 F (y)) g (y)
(see Appendix C).
In a second-price auction, the expected revenue of the seller is

E RII = E [Y2 ]
Z 1
2 3
= y2 (y + 1) 2 (y + 1) dy
0
= 1=3

Therefore, the expected revenues in the two auctions are the same.

2
Problem 2.3 (Stochastic dominance) Consider an N -bidder …rst-price auction with
independent private values. Let be the symmetric equilibrium bidding strategy when
which each bidder’s value is distributed according to F on [0; !] : Similarly, let be
the equilibrium strategy when each bidder’s value distribution is F on [0; ! ] :
a. Show that if F dominates F in termsof the reverse hazard rate (see Appendix
B for a de…nition) then for all x 2 [0; !] ; (x) (x) :
p
2 1
b. By considering F (x) = 3x x on [0; 2 (3 5)] and F (x) = 3x 2x2 on
1
0; 2 , show that the condition that F …rst-order stochastically dominates F is not
su¢ cient to guarantee that (x) (x) :

Solution. Part a. Because G (x) = F (x)N 1 and g (x) = (N 1) F (x)N 2


f (x) ; the
symmetric equilibrium in Proposition 2.2 could be rewritten as follows
Z x
1
(x) = yg (y) dy
G (x) 0
Z x
1
= y (N 1) F (x)N 2 f (x) dy
[F (x)]N 1 0
Z x
f (y)
= (N 1) y dy
F (y)
Z0 x
= (N 1) y (y) dy
0

where (x) is the reverse hazard rate. Similarly, we have


Z x
(x) = (N 1) y (y) dy
0

So it is easy to see that if F dominates F in terms of reverse hazard rate, then


(y) (y) for all y 2 [0; !] : Therefore (x) (x) for all x 2 [0; !].
Part b. Obviously, F (x) F (x), so F stochastically dominates F . The
distributions F and F are illustrated in Figure S2.1, where the solid line represents
F and the dashed line represents F .

3
1.0

0.9

0.8

0.7

0.6

0.5

0.4

0.3

0.2

0.1

0.0
0.0 0.1 0.2 0.3 0.4 0.5
x
Figure S2.1

Suppose there are two bidders, then


Z x
G (y)
(x) = x dy
0 G (x)
Z x
3y y 2
= x dy
0 3x x2
1 2x 9
= x
6 x 3
p
for x 2 0; 12 3 5 : Similarly,
Z x
3y 2y 2
(x) = x dy
0 3x 2x2
1 x
= (8x 9)
6 2x 3
p
for x 2 0; 12 : It is easy to see that (x) < (x) for x 2 (0; 21 3 5 ]: The bidding
strategies and are plotted in Figure S2.2, where is the solid line and is the
dashed line.

4
0.20

0.15

0.10

0.05

0.00
0.0 0.1 0.2 0.3 0.4 0.5
x
Figure S2.2

Problem 2.4 (Mixed auction) Consider an N -bidder auction which is a “mixture”


of a …rst- and second-price auction in the sense that the highest bidder wins and pays
a convex combination of his own bid and the second-highest bid. Precisely, there is a
…xed 2 (0; 1) such that upon winning, bidder i pays bi + (1 ) (maxj6=i bj ) : Find
a symmetric equilibrium bidding strategy in such an auction when all bidders’values
are distributed according to F:

Solution. Suppose all bidders other than 1 follow the strategy : The expected
payo¤ of bidder i from bidding b when his value is x is
1
(b; x)) = G (b) [x b (1 )E [ (Y1 ) j (Y1 ) b1 ]]
" R 1 (b) #
1 (y)g(y)dy
= G (b) x b (1 ) 0
G( 1 (b))
Z 1 (b)
1
= G (b) (x b) (1 ) (y)g(y)dy
0

Maximizing this with respect to b yields the …rst-order condition:


1
g (b) 1
0 = 0 1 (x b) G (b)
(b)
1 1
(1 ) bg( (b)) 0 1
(b)
At a symmetric equilibrium, b = (x), so the …rst-order condition becomes
0 1 1
G (x) (x) + g (x) (x) = xg (x)

5
Using G (x)(1= ) 1 as the integrating factor, the solution to the above di¤erential
equation is easily seen to be
Z x
1
(x) = yg (y) dy
G (x) 0

where G G1= and g = G0 .

Problem 2.5 (Resale) Consider a two-bidder …rst-price auction in which bidders’


values are distributed according to F: Let be the symmetric equilibrium (as derived
in Proposition 2.2). Now suppose that after the auction is over, both the losing and
winning bids are publicly announced. In addition, there is the possibility of post-
auction resale: The winner of the auction may, if he so wishes, o¤ er the object to the
other bidder at a …xed “take-it-or-leave-it” price of p: If the other bidder agrees, then
the object changes hands and the losing bidder pays the winning bidder p. Otherwise,
the object stays with the winning bidder and no money changes hands. The possibility
of post-auction resale in this manner is commonly known to both bidders prior to
participating in the auction. Show that remains an equilibrium even if resale is
allowed. In particular, show that a bidder with value x cannot gain by bidding an
amount b > (x) even when he has the option of reselling the object to the other
bidder.

Solution. First, let us consider the resale stage. Suppose bidder 1 wins the auction
and the announced bids are b1 and b2 . Hence bidder 1 can recover bidder 2’s private
value by x2 = 2 1 (b2 ):Therefore bidder 1 suggests the price x2 which extracts all the
surplus from bidder 2 if x2 x1 , and does not o¤er otherwise. Then bidder 1’s payo¤
is max(x1 b1 ; x2 b1 ). If bidder 1 loses the auction, he gets zero payo¤ because
bidder 2 o¤ers price x1 to him and extracts all the surplus. Rx
1
Second, now we move to the auction stage. Let (x) = F (x) 0 yf (y)dy be the
symmetric equilibrium without resale. We are going to show that any deviation of
bidder 1 from (x) is not pro…table. Suppose bidder 1 deviates by bidding (z) when
his private value is x, while bidder 2 still plays (x2 ). Bidder 1’s ex ante expected
payo¤ is

(x (z)) F (z) if x
Rz z
1 (z; x) =
(x (z)) F (x) + x (y (z)) f (y)dy if x < z

If x2 < z x there is no resale. If z > x x2 ; bidder 1 does not o¤er to bidder


2 and his payo¤ remains the same. If z x2 x, bidder 1 sells to bidder 2 and the

6
payo¤ after resale is x2 (z): Note that
Z z
(x (z)) F (x) + (y (z)) f (y)dy
x
Z z
= xF (x) + yf (y)dy (z)F (z)
x
Z z Z z
= xF (x) + yf (y)dy yf (y)dy
x 0
Z x
1
= xF (x) F (x) yf (y)dy
F (x) 0
= (x (x)) F (x)

so we have

(x (z)) F (z) if x z
1 (z; x) =
(x (x)) F (x) if x < z
which is not more than 1 (x; x). So no deviation strictly increases a bidder’s payo¤
and (x) is still an equilibrium in the presence of resale.

7
3 The Revenue Equivalence Principle
Problem 3.1 (War of attrition) Consider a two-bidder war of attrition in which
the bidder with the highest bid wins the object but both bidders pay the losing bid.
Bidders’ values independently and identically distributed according to F .
a. Use the revenue equivalence principle to derive a symmetric equilibrium bidding
strategy in the war of attrition.
b. Directly compute the symmetric equilibrium bidding strategy and the sellers’
revenue when the bidders’ values are uniformly distributed on [0; 1].

Solution. Part a. Suppose that there is a symmetric, increasing equilibrium of the


war of attrition, , such that the expected payment of a bidder with value 0 is 0.
Since the assumptions of Proposition 3.1 are satis…ed, we must have that for all x,
the expected payment is Z x
m (x) = yf (y) dy
0
On the other hand, we also have

m (x) = E [ (Y1 ) j Y1 < x]


Z x
1
= (y) f (y) dy
F (x) 0
where Y1 is the bid from the other bidder. Combining the two equations, we have
Z x Z x
1
yf (y) dy = (y) f (y) dy
0 F (x) 0
Di¤erentiating both sides with respect to x and rearranging this, we get
Z x
(x) = F (x) x + yf (y) dy
0

Part b. Suppose that bidder 1 has valuation x: He chooses b to maximize his expected
payo¤
Z 1 (b)
1 1
F (b) x 1 (y) f (y) dy
F (b) 0
where the …rst term is the product of his probability of winning and his valuation,
and the second term is his expected payment.
Because F (x) = x and f (x) = 1, the expected payo¤ becomes
Z 1
(b)
1 1
(b) x 1 (y) dy
(b) 0

Maximizing with respect to b yields the …rst-order condition:


1 1 1 1
R 1
(b)
(b) 0 (b) + 0 0 (y) dy
1 ( 1
(b)) ( 1
(b))
0= 0 1
x+ 2
(b) 1
(b)

8
In equilibrium, b = (x) ; and thus the …rst-order condition becomes
Z x
3
x (x) x + (y) dy = 0
0

Di¤erentiating both sides with respect to x, we obtain


0
(x) = 3x

Combining with the initial condition (0) = 0, we can solve the equilibrium bidding
strategy as (x) = 23 x2 :
Therefore the seller’s revenue is
Z Z
2 x 2 x3 2
2E [m (x)] = (y) dy = y dy = 1=3
x 0 x 0 2

Problem 3.2 (Losers-pay auction) Consider a N -bidder losers-pay auction in which


the bidder with the highest bid wins the object and pays nothing while all losing bidders
pays their own bids. Bidders’ valuations independently and identically distributed
according to F .
a. Use the revenue equivalence principle to derive a symmetric equilibrium bidding
strategy in the losers-pay auction.
b. Directly compute the symmetric equilibrium bidding strategy for the case when
the bidders’ values are distributed according to F (x) = 1 e ax over [0; 1).

Solution. Part a. Suppose that there is a symmetric, increasing equilibrium of the


losers-pay auction, , such that the expected payment of a bidder with value 0 is 0.
Since the assumptions of Proposition 3.1 are satis…ed, we must have that for all x,
the expected payment is Z x
m (x) = yg (y) dy
0
On the other hand, we also have

m (x) = (1 G (x)) (x)

which is the product of probability of losing the auction and his own bid. If we
combine the two equations, we have
Z x
yg (y) dy = (1 G (x)) (x)
0

and therefore
Rx
0yg (y) dy
(x) =
1 G (x)
Rx
(N 1) 0 yF N 2 (y) f (y) dy
=
1 FN 1 (x)

9
Part b. Suppose that bidder 1 has valuation x, then he chooses b to maximize
his expected payo¤
1 1
G (b) x 1 G (b) b
where the …rst term is the product of the probability of winning and his valuation,
and the second term is his expected payment.
Maximizing with respect to b yields the …rst-order condition:
!
1 1
g (b) 1 g (b)
0 1 x 1 G (b) 0 1 b=0
(b) (b)
In equilibrium, b = (x), and thus the …rst-order condition becomes
0 g (x) g (x) x
(x) (x) =
1 G (x) 1 G (x)
R x g(y)
Using exp 0 1 G(y) dy as the integrating factor and the initial condition (0) =
0, it is easily seen that
Rx R y g(z) g(y)y
0 exp 0 1 G(z) dz 1 G(y) dy
(x) = R x g(y) (1)
exp 0 1 G(y) dy
Rx
g (y) ydy
= 0
1 G (x)
where the last equality comes from the fact that
Z x Z x
g (y) 1
exp dy = exp dG (y)
0 1 G (y) 0 1 G (y)
Z x
1
= exp d [1 G (y)]
0 1 G (y)
= exp (ln (1 G (x)))
= 1 G (x)
Note that (1) is the same as the equilibrium strategy derived in Part a.
When the bidders’values are distributed according to F (x) = 1 e ax , we have
ax N 1
G (x) = 1 e
ax N 2 ax
g (x) = (N 1) 1 e e a
Therefore the equilibrium bidding strategy in (1) becomes
Rx
g (y) ydy
(x) = 0
1 G (x)
Rx N 2
(N 1) 0 [1 e ay ] e ay aydy
=
1 [1 e ax ]N 1
R ax N 2
(N 1) a1 0 [1 e y ] e y ydy
=
1 [1 e ax ]N 1

10
4 Quali…cations and Extensions
Problem 4.1 (Risk-averse bidders) There are two bidders with private values which
are distributed independently according to the uniform distribution F (x) = x over
p
[0; 1] : Both bidders are risk-averse and have utility functions u (z) = z: Find sym-
metric equilibrium bidding strategies in a …rst-price auction.

Solution. This is a special case of Example 4.1 with a = 1=2 and F (x) = x, so
Z x Z x
1 1 2
(x) = yf (y) dy = 2 y2ydy = x
F (x) 0 x 0 3

Problem 4.2 (Increase in risk aversion) Consider an N -bidder …rst-price auction


where each bidder’s value is distributed according to F: All bidders are risk averse with
a utility function u that satis…es u(0) = 0; u0 > 0; u00 < 0: Show that if one changed
the utility function of all bidders from u (z) to (u (z)), where is an increasing and
concave function satisfying (0) = 0; then this would lead to a higher symmetric
equilibrium bidding strategy.

Solution. Let be the equilibrium bidding strategy for utility u while ^ be the
equilibrium bidding strategy for (u). From equation (4.2), we have

0 u (x (x)) g (x)
(x) =
u0 (x (x)) G (x)

(u (x ^ (x))) g (x)
^ 0 (x) = 0
(u (x ^ (x))) u0 (x ^ (x)) G (x)
0
Notice that is strictly concave and (0) = 0, for all y > 0, (y) = (y) > y.
Using this fact, for x > 0 we have

(u (x ^ (x))) g (x)
^ 0 (x) = 0
(u (x ^ (x))) u0 (x ^ (x)) G (x)
u (x (x)) g (x)
> = 0 (x)
u0 (x (x)) G (x)

or equivalently
^ 0 (x) > 0
(x) for x > 0 (2)
It is also easy to check that
^ (0) = (0) (3)
Equations (2) and (3) imply that for all x > 0;

^ (x) > (x)

11
Problem 4.3 (Asymmetric …rst-price auction) Suppose that bidder 1’s value X1 is
distributed according to F1 (x) = 14 (x 1)2 over [1; 3] and bidder 2’s value is distrib-
uted according to F2 (x) = exp 23 x 2 over [0; 3] :
a. Show that 1 (x) = x 1 and 2 (x) = 23 x constitute equilibrium bidding strate-
gies in a …rst-price auction.
b. Compare the expected revenues in a …rst- and second-price auction.

Solution. Part a. Given bidder 2’s strategy 2 (x) = 23 x, bidder 1 with valuation
x chooses b to maximize his expected payo¤
1
F2 2 (b) (x b) = exp (b 2) (x b)

The …rst-order condition is

exp (b 2) (x b) exp (b 2) = 0

In equilibrium, b = 1 (x), and thus the …rst-order condition yields

exp ( 1 (x) 2) [x 1 1 (x)] =0

It is easy to see that 1 (x) = x 1 satis…es the equation above, and it is easy to see
that the marginal payo¤ is negative if 1 (x) > x 1 and positive if 1 (x) < x 1;
therefore it is the best response to bid 1 (x) :
On the other hand, given bidder 1’s strategy 1 (x) = x 1 , bidder 2 with
valuation x chooses b to maximize his expected payo¤

1 1
F1 1 (b) (x b) = b2 (x b)
4
The …rst-order condition is
1 1 2
b (x b) b =0
2 4
In equilibrium, b = 2 (x), and thus the …rst-order condition yields

3 2
2 (x) x 2 (x) =0
4 3

It obvious that 2 (x) = 32 x satis…es the equation above, and it is easy to see that the
marginal payo¤ is negative if 2 (x) > 32 x and positive if 2 (x) < 23 x; therefore it is
the best response to bid 1 (x) :
In sum, given bidder 1’s strategy 1 (x) ; bidder 2’s best response is 2 (x), and
given bidder 2’s strategy 2 (x) bidder 1’s best response is 1 (x), therefore 1 (x)
and 2 (x) constitute an equilibrium.
Part b. In the …rst-price auction, the expected revenue is
1 1
E F2 2 ( 1 (x1 )) 1 (x1 ) + E F1 1 ( 2 (x2 )) 2 (x2 )

12
where the …rst term is the expected payment from bidder 1 and the second term is
the expected payment from bidder 2. Given the distributions and bidding strategies,
the expected revenue could be rewritten as
Z 3
E R I
= F2 2 1 ( 1 (x1 )) 1 (x1 ) dF1 (x1 )
1
Z 3
+ F1 1 1 ( 2 (x2 )) 2 (x2 ) dF2 (x2 )
0
Z
1 3
= exp (x1 3) (x1 1)2 dx1
2 1
Z 3
4 2
+ exp x2 2 x32 dx2
81 0 3
= 1: 567 7

In the second-price auction, it is a dominant strategy to bid one’s value. We


can either use the same method for the …rst price auction or calculate the expected
revenue using price distribution, and here we use the latter. The distribution of price
is 8
<exp( 2p 2) for p < 1
3
L(p) = p 2
2
2p p 2
2
2p
: + exp( 3 2) exp( 3 2) for 1 p 3:
2 2

So the expected revenue in the second-price auction is


Z 1
II 2p
E R = p d exp( 2)
0 3
Z 3 !
p 2 2 2p p 2 2 2p
+ pd + exp( 2) exp( 2)
1 2 3 2 3
Z 3 Z 3 !
2p p 2 2
= p d exp( 2) + pd
0 3 1 2
Z 3 !
p 2 2 2p
pd exp( 2)
1 2 3
Z 3 Z 3
11 1 4 2p p 2 2
= + exp( ) exp( 2) dp dp
4 4 3 0 3 1 2
Z 3
p 2 2 2p
+ exp( 2)dp
1 2 3
= 1: 449 5

where the third equality comes from integration by parts.


Thus, E RI > E RII .

Problem 4.4 (Equilibrium with reserve price) Suppose that bidder 1’s value X1 is
distributed uniformly on [0; 2] while bidder 2’s value X2 is distributed uniformly on

13
[ 32 ; 52 ]. The object is sold via a …rst-price auction with a reserve price r = 1. Verify
that 1 (x) = x2 + 21 and 2 (x) = x2 + 14 constitute equilibrium strategies.

Solution. F1 (x) = x2 for x 2 [0; 2] and F2 (x) = x 23 for x 2 32 ; 52 . Given bidder


2’s strategy 2 (x) = x2 + 41 , bidder 1 with valuation x chooses b to maximize his
expected payo¤. If b r, the expected payo¤ is
1
F2 2 (b) (x b)
= (2b 2) (x b) =2

and the corresponding marginal payo¤ is

x 2b + 1

The …rst-order condition is


x 2b + 1 = 0
At equilibrium, b = 1 (x), and thus the …rst-order condition yields

x 2 1 (x) +1=0

It is easy to see that 1 (x) = x+1


2 satis…es the equation above. Moreover, it is easy to
see that the marginal payo¤ is negative if b > x+1
2 and the marginal payo¤ is positive
x+1 x+1
if b < 2 : Therefore 1 (x) = 2 is the best response to 2 (x) :
On the other hand, given bidder 1’s strategy 1 (x) = x2 + 21 , bidder 2 with
valuation x chooses b to maximize his expected payo¤. If b r, The expected payo¤
is
F1 1 1 (b) (x b) = (2b 1) (x b) =2
The …rst-order condition is

2 (x b) (2b 1) = 0

At equilibrium, b = 2 (x), and thus the …rst-order condition yields

2x 4 2 (x) +1=0

It obvious that 2 (x) = x2 + 14 satis…es the equation above, and it is bidder 2’s best
response to 1 (x) by the same reasoning in the case of bidder 1.
We have shown that given bidder 1’s strategy 1 (x) bidder 2’s best response
is 2 (x), and given bidder 2’s strategy 2 (x) bidder 1’s best response is 1 (x).
Therefore 1 and 2 constitute an equilibrium.

Problem 4.5 (Discrete values) Suppose that there is no uncertainty about bidder 1’s
value and X1 = 2 always. Bidder 2’s value, X2 , is equally likely to be 0 or 2.
a. Find equilibrium bidding strategies in a …rst-price auction. (Note that since
values are discrete, the equilibrium will be in mixed strategies.)
b. Compare the revenues in a …rst- and second-price auction.

14
Solution. Part a. Note that when bidder 2 has value 0, it is his weakly dominant
strategy to bid zero. Let us assume bidder 1’s mixed-strategy has cumulative dis-
tribution M1 (b) and bidder 2 with valuation 2 has mixed-strategy with cumulative
distribution M2 (b). We only consider the case that M1 and M2 are di¤erentiable.
If bidder 1 bid b, his expected payo¤ is

1 1
+ M2 (b) (2 b)
2 2

which is the product between the probability of winning and the payo¤ when he wins.
Because bidder 1 plays mixed-strategy, his expected payo¤ should be the same for
any b in the support of M1 ; therefore

1 1
+ M2 (b) (2 b) = C
2 2

where C is a constant. So we have


2C
M2 (b) = 1 (4)
2 b
It should also be true that
M2 (0) = 0 (5)
(4) and (5) imply that
b
M2 (b) = ; for b 2 [0; 1]
2 b
If bidder 2 with valuation 2 bids b, this expected payo¤ is

M1 (b) (2 b)

which is the product between the probability of winning and the payo¤ when he wins.
Similar to the analysis for bidder 1, we have

C0
M1 (b) = (6)
2 b
for some constant C 0 .
Because every bidder does not bid higher than his opponent, the highest bid is
the same for both bidders. Note that the highest bid for bidder 2 is 1, so we have

M1 (1) = 1 (7)

(6) and (7) imply that


1
M1 (b) = , for b 2 [0; 1]
2 b
where 0 is a mass point of the distribution of M1 :

15
Part b. When X2 = 0, the cumulative distribution for price is M1 ; and when
X2 = 2, the cumulative distribution for price is M1 M2 , so the expected revenue in
the …rst-price auction is

E RI = Pr (X2 = 0) E [P j X2 = 0] + Pr (X2 = 2) E [P j X2 =]
Z Z
1 1 1 1
= bdM1 (b) + bd [M1 (b) M2 (b)]
2 0 2 0
Z 1 Z 1
1 1 b
= bd + bd
2 0 2 b 0 (2 b)2
Z 1 Z 1
1 1 b
= 1 db + 1 db
2 0 2 b 0 (2 b)2
1
=
2
where the fourth equality comes from integration by parts.
The expected revenue in the second-price auction is
1 1
E RII = 0+ 2=1
2 2
So we have E RI < E RII :

16
5 Mechanism Design
Problem 5.1 (Surplus extraction) Show that if buyers’values are independently dis-
tributed, then the seller cannot design an incentive compatible and individually ratio-
nal mechanism that extracts the whole surplus from buyers. (In doing this problem,
use only the results of Section 5.1 and not those from Section 5.2.)

Solution. Suppose that there exists a mechanism extracting all the surplus from
the buyers. We also assume that the supports of the buyer value distributions are
…nite [ai ; bi ]. Using revelation principle we may say then there should be a direct
mechanism which extracts the whole surplus from the buyers. Using the notation of
Section 5.1 the fact that the seller extracts the whole surplus from the buyers means
that for all i and for all xi , Ui (xi ) = 0:
Because the buyers’values are independently distributed, when the buyer reports
zi when his true value is xi , his expected payo¤ is (see Section 5.1)

qi (zi )xi mi (zi )

and
Ui (xi ) = qi (xi )xi mi (xi )
Incentive compatibility implies that

Ui (xi ) qi (zi )xi mi (zi )


= qi (zi )zi mi (zi ) + qi (zi ) (xi zi )
= Ui (zi ) + qi (zi ) (xi zi )

Because the seller extracts all the surplus from all the buyers, Ui (xi ) = Ui (zi ) = 0
and for all i and for all xi and zi ;

qi (zi ) (xi zi ) 0

The inequality above implies qi (zi ) (bi zi ) 0; therefore qi (zi ) = 0 if zi < bi .


So qi (zi ) = 1 only if zi is the highest value. Note that the mechanism is incentive
compatible, so in equilibrium xi = zi ; so the seller allocates the object only when
one of the buyers has the highest value. Then the seller does not extract the whole
surplus when every buyer’s value is below the highest one. This contradicts our initial
assumption.

Problem 5.2 (Optimal auction) There is a single object for sale and there are 2
potential buyers. The value assigned by buyer 1 to the object X1 is uniformly drawn
from the interval [0; 1 + k] whereas the value assigned by buyer 2 to the object X2
is uniformly drawn from the interval [0; 1 k] ; where k is a parameter satisfying
0 k < 1. The two values X1 and X2 are independently distributed.
a. Suppose the seller decides to sell the object using a second-price auction with
a reserve price r: What is the optimal value of r and what is the expected revenue of
the seller?
b. What is the optimal auction associated with this problem?

17
x1
Solution. Part a. Let the cumulative distribution for X1 be F1 (x1 ) = 1+k and
x2
the cumulative distribution for X2 be F2 (x2 ) = 1 k . The corresponding density
1
functions are f1 (x1 ) = 1+k and f2 (x2 ) = 1 1 k . So the expected payment from buyer
1 is
Z 1+k Z 1+k Z x1
1
[m1 (x1 ; r)] f1 (x1 ) dx1 = rF2 (r) + yf2 (y) dy dx1
r r r 1+k
Z 1+k Z x1
r 1 1
= r + y dy dx1
r 1 k r 1 k 1+k

where the …rst equality comes from (2.9) and G = F2 .


The payment from buyer 2 is

Z 1 k Z 1 k Z x2
1
[m2 (x2 ; r)] f2 (x2 ) dx2 = rF1 (r) + yf1 (y) dy dx2
r r r 1 k
Z 1 k Z x2
r 1 1
= r + y dy dx2
r 1+k r 1+k 1 k

The optimal reserve price r maximizes the expected revenue below


Z 1+k Z x1
r 1 1
E [m1 (X1 ; r)] + E [m2 (X2 ; r)] = r + y dy dx1
r 1 k r 1 k 1+k
Z 1 k Z x2
r 1 1
+ r + y dy dx2
r 1+k r 1+k 1 k
1
= 3k 2 4r3 + 3r2 + 1
3 (k 2 1)

Di¤erentiating this with respect to r, we obtain

12r2 + 6r = 0
1
So we have r = 2 and the corresponding expected revenue for the seller is

1 5
3k 2 +
3 (1 k2 ) 4

Part b. The virtual valuation for buyer 1 with value x1 is

1 F1 (x1 )
1 (x1 ) = x1 = 2x1 (1 + k)
f1 (x1 )

The virtual valuation for buyer 1 with value x1 is

1 F2 (x2 )
2 (x2 ) = x2 = 2x2 (1 k)
f2 (x2 )

18
Therefore the smallest value for bidder 1 that wins against x2 is

y1 (x2 ) = inf fz1 : 1 (z1 ) 0 and 1 (z1 ) 2 (x2 )g


= inf fz1 : 2z1 (1 + k) 0 and 2z1 (1 + k) 2x2 (1 k)g
1+k
= max ; x2 + k
2

and

y2 (x1 ) = inf fz2 : 2 (z2 ) 0 and 2 (z2 ) 1 (x1 )g


= inf fz2 : 2z2 (1 k) 0 and 2z2 (1 k) 2x1 (1 + k)g
1 k
= max ; x1 k
2

Because 1 and 2 are increasing functions, the design problem is regular and by
Proposition 5.3, the optimal mechanism (Q; M ) is given as
1+k
1 if x1 x2 > k and x1 2
Q1 (x1 ; x2 ) =
0 otherwise
1 k
1 if x1 x2 < k and x2 2
Q2 (x1 ; x2 ) =
0 otherwise
For i = 1; 2,
yi (x i ) if Qi (x1 ; x2 ) = 1
Mi (x1 ; x2 ) =
0 if Qi (x1 ; x2 ) = 0

Problem 5.3 (Dissolving a partnership) Two agents jointly own a …rm and each has
an equal share. The value of the whole …rm to each is a random variable Xi which
is independently and uniformly distributed on [0; 1] : Thus in the current situation,
agent 1 derives a value 21 X1 from the …rm and agent 2 derives a value 12 X2 from the
…rm. Suppose that the two agents wish to dissolve their partnership and since the
…rm cannot be subdivided, ownership of the whole …rm would have go to one of the
two agents.
a. Consider the following procedure for reallocating the …rm. Both agents bid
amounts b1 and b2 and if bi > bj , then i gets ownership of the whole …rm and pays
the other agent j the amount bi . Find a symmetric equilibrium bidding strategy in
this auction.
b. Is the procedure outlined above e¢ cient? Is it individually rational?
c. Calculate each agent’s payments in the VCG mechanism associated with this
problem.

Solution. Part a. Let the symmetric equilibrium bidding strategy be (x). Given
1
the uniform distribution, (bi ) is the probability of winning for bidder i when he

19
bids bi ; and his expected payo¤ is

1 1 1
(bi ) xi bi + 1 (bi ) E [ (X i ) j bi ; (X i ) > bi ]
2
Z 1
1 1 1 (y)
= (bi ) xi bi + 1 (bi ) 1 dy
2 1
(bi ) 1 (bi )
Maximizing this respect to bi yields the …rst-order condition
1 1 1 1 1
0 1 xi bi (bi ) (bi ) 0 1 =0
(bi ) 2 (bi )
At symmetric equilibrium bi = (xi ) so the …rst-order condition becomes
1 1 1
0 xi (xi ) xi (xi ) 0 =0
(xi ) 2 (xi )

0 2 1
(xi ) + (xi ) =
xi 2
Using the integrating factor x2i , we solve the equation above and have the solution
(x) = 16 x which is the symmetric equilibrium bidding strategy.
Part b. Because the bidding function is strictly increasing, the buyer with a
higher valuation wins the …rm, therefore the procedure above is e¢ cient.
The expected payment for buyer i with value xi is
Z 1
(y)
mi (xi ) = xi (xi ) (1 xi ) dy
xi 1 xi
Z 1
1 2 1
= xi ydy
6 xi 6
1 2 11
= x 1 x2i
6 i 62
1 1
= + x2i
12 4
so mi (0) < 0 therefore the procedure is individually rational.
Part c. Using formula (5.22), the payments in VCG mechanism are

MiV (xi ; x i ) = W ( i ; x i ) W i (xi ; x i )


1
= x i W i (xi ; x i )
2
1
= 2 x i if xi > x i
0 if xi < x i
where the second equality comes from
0 if xi > x i
W i (xi ; x i ) = 1
2x i if xi x i

20
Problem 5.4 (Negative externality) The holder of a patent on a cost reducing process
is considering the possibility of licensing it to one of two …rms. The two …rms are
competitors in the same industry and so if …rm 1 obtains the license, its pro…ts will
increase by X1 while those of …rm 2 will decrease by X2 , where is a known
parameter satisfying 0 < < 1: This is because if …rm 1 gets the license, …rm 2 will
have a cost disadvantage relative to …rm 1. Similarly, if …rm 2 obtains the license, its
pro…ts will increase by X2 while those of …rm 1 will decrease by X1 . The variables
X1 and X2 are uniformly and independently distributed on [0; 1] : Firm 1 knows the
realized value x1 of X1 and only that X2 is uniformly distributed, and similarly for
…rm 2.
a. Suppose that the license will be awarded on the basis of a …rst-price auction.
What are the equilibrium bidding strategies? What is the expected revenue of the
seller, that is, the holder of the patent?
b. Find the payments in the VCG mechanism associated with this problem. Are
the expected payments the same as in a …rst-price auction?
c. Suppose that the patent holder is a government laboratory and it wants to
ensure that the license is allocated e¢ ciently and that the net payments of the buyers
add up to zero, that is, the “budget” is balanced. What is the associated “expected
externality” mechanism for this problem? Is it individually rational? Does there
exist an e¢ cient, incentive compatible, and individually rational mechanism that also
balances the budget?

Solution. Part a. Let the symmetric equilibrium bidding strategy be (x) : If the
…rm i with value xi bids bi , his expected payo¤ is
1 1
(bi ) [xi bi ] + 1 (bi ) ( xi )

Maximizing this respect to bi yields the …rst-order condition


1 1 xi
0 1 [xi bi ] (bi ) + 0 1 =0
(bi ) (bi )
At symmetric equilibrium bi = (xi ) so the …rst-order condition becomes
1 xi
0 [xi (xi )] xi + 0 =0
(xi ) (xi )
which could be rewritten as
0 1
(xi ) + (xi ) = 1 +
xi
Therefore the equilibrium bidding strategy is
1+
(x) = x
2
The payment from bidder i with valuation xi is
1+
mi (xi ) = xi (xi ) = x2
2

21
so the seller’s revenue is
Z 1
I 1+ 1+
E R = 2E [mi (Xi )] = 2 x2i dxi =
0 2 3
Part b. The VCG payments in this problem are

MiV (xi ; x i ) = W (0; x i ) W i (xi ; x i )


= x i W i (xi ; x i )
x i if xi > x i
=
0 if xi < x i

where the last equality comes from

0 if xi > x i
W i (xi ; x i ) =
x i if xi x i

So the seller’s revenue from the VCG mechanism is


Z 1 Z xi
y 1
E RV = 2E [Xi E [Xj j Xi ; Xj < Xi ]] = 2 xi dydxi =
0 0 xi 3

Therefore, we have E RI > E RV


Part c. As in Section 5.3, the “expected externality”mechanism for this problem
Q ; M A is de…ned by

MiA (xi ; x i ) = EXi [Wi (Xi ; x i )] EX i [W i (xi ; X i )]


Z 1 Z 1
= xi dxi x i dx i
x i xi
1 2
= x x2 i
2 i
where the second equality comes from

Xi if Xi > x i
Wi (Xi ; x i ) =
0 if Xi < x i
0 if xi > X i
W i (xi ; X i ) =
X i if xi < X i

Note that
Z 1 Z 1
1 1
mi (0) = MiA (0; x i ) dx i = 0 x2 i dx i = 0
0 0 2 6
so it is individual rational.
Because VCG mechanism derived in Part b results an expected surplus, Proposi-
tion 5.6 guarantees the existence of an e¢ cient, incentive compatible, and individually
rational mechanism that balances the budget.

22
Problem 5.5 (A non-standard selling method) There is a single object for sale and
there are two interested buyers. The values assigned by the buyers to the object are
independently and uniformly distributed on [0; 1] : As always, each buyer knows the
value he or she assigns to the object but the seller knows only that each is inde-
pendently and uniformly distributed. The seller assigns a value of 0 to the object.
Suppose that the seller adopts the following selling strategy. She approaches one of
the buyers (chosen at random) and makes a “take-it or leave-it” o¤ er at a …xed price
p1 . If the …rst buyer accepts the o¤ er, the object is sold to him at the o¤ ered price.
If the …rst buyer declines the o¤ er, the seller then approaches the other buyer with
a “take-it or leave-it” o¤ er at a …xed price p2 . If the second buyer accepts the o¤ er,
the object is sold to him at the o¤ ered price. If neither buyer accepts then the seller
keeps the object.
a. What are the optimal values of p1 and p2 ?
b. What is the expected revenue to the seller if he adopts this selling scheme?
c.How does it compare to a standard auction? In particular, does the revenue
equivalence principle apply?.

Solution. Part a. If the seller choose p1 and p2 , his expected revenue is

(1 p1 ) p1 + p1 (1 p 2 ) p2

Maximizing this respect to p1 and p2 yields the …rst-order conditions

1 2p1 p2 p22 = 0
p1 (1 2p2 ) = 0

If p1 = 0, the expected revenue is 0, so it can not be a solution. Therefore


3
p1 =
8
1
p2 =
2
Part b. The expected revenue is
3 3 3 1 1
(1 p1 ) p1 + p1 (1 p 2 ) p2 = 1 + 1
8 8 8 2 2
21
=
64
Part c. From Example 3.1, we know that the standard auctions has expected
revenue 13 which is more than 21
64 . The selling scheme could also be viewed as a direct
mechanism (Q; M ) where

1 if x1 p1
Q1 (x1 ; x2 ) =
0 if x1 < p1

1 if x1 < p1 and x2 p2
Q2 (x1 ; x2 ) =
0 if x1 p1 or x2 < p2

23
For i = 1; 2 ;
pi if Qi (x1 ; x2 ) = 1
Mi (x1 ; x2 ) =
0 if Qi (x1 ; x2 ) = 0
However the mechanism does not have the same allocation rule as standard auc-
tions. For example, If x2 > x1 > p1 ; buyer 1 gets the object, while in standard
auction buyer 2 should get the object. As a result, the revenue equivalence principle
does not apply here.

24
6 Auctions with Interdependent Values
Problem 6.1 (A¢ liation) Suppose there are two bidders who receive private signals
X1 and X2 which are jointly distributed over the set
n p o
S = (x1 ; x2 ) 2 [0; 1]2 : x1 x2 (x1 )2

with a uniform density. The bidders attach a common value V = 21 (X1 + X2 ) to the
object.
a. Find symmetric equilibrium bidding strategies in both a …rst-price and a second-
price auction?
b. Calculate the expected revenues from both auctions and show that the revenue
in a second-price auction is greater than that in a …rst-price auction.

Solution. Part a. First, we apply Proposition 6.3 to …nd the equilibrium strategy
for the …rst-price auction.
The area of the set S is
Z 1
p 1
x1 x21 dx1 =
0 3

so the density function of X1 and X2 is f (x1 ; x2 ) = 3. Let G ( j x) be the distribution


of X2 conditional on X1 = x and let g ( j x) be the associated conditional density
function, then we have
Ry
x2 f (s; x)ds y x2
G (y j x) = R p = p
x
2 f (s; x)ds
x x2
x
f (y; x) 1
g (y j x) = R px =p
f (s; x)ds x x2
x2

Hence,
g(x j x) 1
=
G(x j x) x x2
Therefore,
Z x
g(t j t)
L(y j x) = exp dt
y G(t j t)
Z x
1
= exp dt
y t t2
Z x
1 1
= exp + dt
y t 1 t
= exp ln t jxy + ln(1 t) jxy
1 x y
=
x 1 y

25
Proposition 6.3 implies that the symmetric equilibrium strategies for the …rst-price
auction are given by
Z x
I
(x) = v(y; y)dL(y j x)
0
Z x
1 x y
= yd
0 x 1 y
Z x
1 x x2 1
= 1 dy
x 1 x 0 1 y
1 x x2 1 x
= ( ln (1 x) x)
x 1 x x
1 x
= 1+ ln(1 x)
x
Second, Proposition 6.1 implies the symmetric equilibrium strategies in the second-
price auction are given by
II x+x
(x) = v(x; x) = =x
2
Part b. Let us …rst consider the …rst-price auction. The expected payment from
bidder i with value x is the product of his bid and his probability of winning
mI (x) = I
(x) G (x j x)
1 x x x2
= 1+ ln(1 x) p
x x x2
So the expected revenue for the seller is
E RI = 2E mI (Xi )
Z 1
= 2 mI (x) fi (x) dx
0
Z 1
p
= 6 mI (x) ( x x2 )dx
0
Z 1 Z 1
2
= 6 (x x )dx + (1 x)2 ln(1 x)dx
0 0
1
=
3
where the third equality comes from the fact that the density of the marginal distri-
bution of Xi is
Z px Z px
p
fi (x) = f (t; x) dt = 3dy = 3( x x2 )
x2 x2

In the second-price auction, the expected payment from the bidder with value x
is
x x2
mII (x) = II
(x)G(x j x) = x p
x x2

26
So the corresponding expected revenue for the seller is
E[RII ] = 2E mII (Xi )
Z 1
= 2 mII (x)fi (x)dx
0
Z 1
= 6 x(x x2 )dx
0
1
=
2
Thus, E RI < E RII .
Problem 6.2 (Lack of a¢ liation) Suppose there are two bidders whose private values
X1 and X2 which are jointly distributed over the set
n o
S = (x1 ; x2 ) 2 [0; 1]2 : x1 + x2 1

with a uniform density. Show that a …rst-price auction with this information structure
does not have a monotonic pure strategy equilibrium.
Solution. This proof is taken from Section 6 in Reny and Zamir (2004). Suppose
to the contrary that such an equilibrium exists, and denote the equilibrium bidding
strategies as 1 ( ) and 2 ( ).
Let us …rst show that x2 2 (x2 ) for all x2 2 [0; 1). Suppose not and there
exists some x^2 2 [0; 1) such that 2 (^ x2 ) > x^2 . Since 2 is nondecreasing, 2 (x2 ) > x2
for all x2 2 (^ x2 ; 2 (^
x2 )). Because bidders do not bid above their highest value,
2 (^
x2 ) 1: Consider the event fx1 < x2 ; x2 2 (^ x2 ; 2 (^
x2 ))g. First, this event has
positive probability. Second, if bidder 1 wins the object in this event, his bid 1 (x1 )
satis…es
1 (x1 ) > 2 (x2 ) > x2 > x1
therefore bidder 1 has negative payo¤. Third, if bidder 2 wins the object, he also has
negative payo¤ because 2 (x2 ) > x2 . So there is a positive probability that at least
one of the players has negative payo¤, which contradicts equilibrium. As a result,
xi i (xi ) for i = 1; 2:
For any " 2 (0; 1), suppose bidder i has value 1 ". He knows that the other
bidder’s value is below ", so are the other bidder’s bids. Thus i’s bid should be
no more than ", so i (1 ") " for any " 2 (0; 1). Therefore, the only possible
nondecreasing strategy equilibrium is 1 (x1 ) = 2 (x2 ) = 0 for all (x1 ; x2 ) in S, which
is clearly not an equilibrium.
Problem 6.3 (Bidding gap) Consider a common value second-price auction with
two bidders. The bidders receive signals X1 and X2 , respectively, and these are in-
dependently and uniformly distributed on [0; 1]. Each bidder’s value for the object is
V = 21 (X1 + X2 ).
a. If the seller sets a reserve price r > 0; show that there is no bid in the neigh-
borhood of the reserve price.
b. Find the optimal reserve price.

27
Solution. Part a. Let the symmetric equilibrium bidding function be ( ). If
bidder 1 with value x bids b r , his expected payo¤ is
Z 1
(b)
1
(b; x) = v (x; y) g (y j x) dy rG (r) j x
0
Z 1
(b)
(y) g (y j x) dy
1
(r)
Z 1
(b) Z 1
(b)
x+y 1
= dy r (r) (y) dy
0 2 1
(r)

Maximizing it with respect to b yields the …rst-order condition


1
x+ (b) 1 1
(b) 0 1 =0
2 (b)

At symmetric equilibrium, b = (x) so the …rst-order condition becomes

x+x 1
(x) 0 =0
2 (x)
So the equilibrium bidding strategy is

(x) = x

The corresponding payo¤ is


Z x Z x
x+y 2
( (x) ; x) = dy r ydy
0 2 r
1 2
= x 2r2
4
p
which means p that ( (x) ; x) 0 only when x 2r. Therefore only bidders with
values in 2r; 1 will participate in the auction, and there is a gap between reserve
p
price r and the lowest bid 2r.
Part b. Suppose the object has no value to the seller. The expected revenue for
the seller from setting a reserve price r > 0 is
Z 1
E [R] = 2 p xdy = 1 2r2
2r

So the optimal reserve price is 0.

Problem 6.4 (Common value auction with Pareto distribution) Suppose that two
bidders are competing in a …rst-price auction for an object with common value V;
which is distributed according to a Pareto distribution FV (v) = 1 v 2 over [1; 1):
Prior to bidding, each bidder receives a signal Xi whose distribution, conditional on
the realized common value V = v, is uniform over [0; v] ; that is, FXi jV (xi j v) =

28
(xi =v) : Conditional on V = v; the signals X1 and X2 are independently distributed.
Verify that the following strategy constitutes a symmetric equilibrium of the …rst-price
auction
(x) = 32 max fx; 1g (1 + max fx; 1g 2 )

Solution. Part a. Let the symmetric equilibrium bidding function be ( ). If


bidder 1 with value x bids b r , his expected payo¤ is
Z 1
(b)
1
(b; x) = v (x; y) g (y j x) dy rG (r) j x
0
Z 1
(b)
(y) g (y j x) dy
1
(r)
Z 1
(b) Z 1
(b)
x+y 1
= dy r (r) (y) dy
0 2 1
(r)

Maximizing with respect to b yields the …rst-order condition


1
x+ (b) 1 1
(b) 0 1 =0
2 (b)

At a symmetric equilibrium, b = (x) so the …rst-order condition becomes

x+x 1
(x) 0 =0
2 (x)

So the equilibrium bidding strategy is

(x) = x

The corresponding payo¤ is


Z x Z x
x+y 2
( (x) ; x) = dy r ydy
0 2 r
1 2
= x 2r2
4
p
which means p that ( (x) ; x) 0 only when x 2r. Therefore only bidders with
values in 2r; 1 will participate in the auction, and there is a gap between reserve
p
price r and the lowest bid 2r.
Part b. Suppose the object has no value to the seller. The expected revenue for
the seller from setting a reserve price r > 0 is
Z 1
E [R] = 2 p xdy = 1 2r2
2r

So the optimal reserve price is 0.

29
The density function of joint distribution of X1 ; X2 ; V is
h (x1 ; x2 ; v) = fX1 jV (x1 j v) fX2 jV (x2 j v) fV (v)
5
= 2v for 0 x1 ; x2 v.
so the the density of joint distribution of X1 and X2 is
Z +1
h (x1 ; x2 ) = h (x1 ; x2 ; v) dv
maxfx1 ;x2 ;1g
Z +1
5
= 2v dv
maxfx1 ;x2 ;1g
1
max fx1 ; x2 ; 1g 4
=
2
and the density function of V conditional on X1 = x1 and X2 = x2 is
h (x1 ; x2 ; v) 4v 5
h (v j x1 ; x2 ) = = 4
h (x1 ; x2 ) max fx1 ; x2 ; 1g
for v max fx1 ; x2 ; 1g
The distribution of X2 given X1 = x1 is
Z x2 Z x2
1 4
G (x2 j x1 ) = h (x1 ; t) dt = max fx1 ; t; 1g dt
0 0 2
1 4
= 2 x2 max fx1 ; 1g if x2 max fx1 ; 1g
2 3 1 3
3 max fx1 ; 1g 6 x2 if x2 > max fx1 ; 1g
and its density function is
1 4
2 max fx1 ; 1g if x2 max fx1 ; 1g
g (x2 j x1 ) = 1 4
2 x2 if x2 > max fx1 ; 1g
and we also have
v (x; y) E [V j X1 = x; X2 = y]
Z +1
= vh (v j x; y) dv
maxfx;y;1g
Z +1
4v 5
= v 4 dv
maxfx;y;1g max fx; y; 1g
4
= max fx; y; 1g
3
Note that v (y; y) is no longer strictly increasing in y and (x) is not strictly
increasing neither, so the proof of Proposition 6.3 does not apply to this question.
As a result, we are not going to use Proposition 6.3 to derive the equilibrium, but
it is easy to check that Proposition 6.3 also gives the same equilibrium, so it is still
true in this context. v (y; y) is illustrated in Figure S6.1 where the horizontal axis
represents y and the vertical axis represents v (y; y). (x) is illustrated similarly in
Figure S6.2.

30
4

0
0.0 0.5 1.0 1.5 2.0 2.5 3.0 3.5 4.0
y
Figure S6.1

0
0.0 0.5 1.0 1.5 2.0 2.5 3.0 3.5 4.0
x
Figure S6.2

31
First, if a bidder with value x > 1 bids (z) where z > x; his expected payo¤ is
Z z
(z; x) = [v (x; y) (z)] g (y j x) dy
0
Z z
4 2z
= max fx; yg 1 + z 2 g (y j x) dy
3 3
Z0 x
4 2z 1 4
= x 1+z 2 x dy
0 3 3 2
Z z
4 2z 1 4
+ y 1+z 2 y dy
x 3 3 2
4 3 1 1 2 1
= x 2 x z+ + 2
+ 4
9 z 9 z z

It is easy to see that

@ (z; x) 4 1 1 1
= 1 <0
@z 9 z2 z3 x3

So we have
(z; x) < (x; x) for z > x > 1 (8)
Similarly, if bidder 1 with value x > 1 bids (z) where 1 < z < x, his expected
payo¤ is
Z z
4 2z
(z; x) = max fx; yg 1 + z 2 g (y j x) dy
3 3
Z0 z
4 2z 1 4
= x 1+z 2 x dy
0 3 3 2
2 3 z2 1
= x z 1+ 2 x 4
3 3 z
so
@ (z; x) 2 4
= x (x z) > 0
@z 3
Then we have
(z; x) < (x; x) for 1 < z < x (9)
If bidder 1 with value x > 1 bids (z) where 0 z < 1, he can only win the
object in a tie and his expected payo¤ is
Z
1 1 4 4 1 4
(z; x) = x x dy
2 0 3 3 2
1 1 1 1
= 3
3x 3 x4
1 1 1 1
< 2
3x 3 x4
= (x; x)

32
so
(z; x) < (x; x) for 0 z<1<x (10)
Therefore (8), (9) and (10) imply that the bidder with value x > 1 bids (x).
Second, let us consider the bidder with value 0 x 1. If he bids (z) where
z 2 [0; 1], his expected payo¤ is
Z
1 1 4 4 1
(z; x) = dy = 0
2 0 3 3 2
so
(z; x) = (x; x) for 0 z 1 and 0 x 1 (11)
If the bidder bids (z) where z > 1, his expected payo¤ is
Z z
2z
(z; x) = v (x; y) 1 + z 2 g (y j x) dy
0 3
Z 1
4 2z 1
= 1+z 2 dy
0 3 3 2
Z z
4 2z 1 4
+ y 1+z 2 y dy
1 3 3 2
4 1 1 2 1
= 1 z+ + 2
+ 4
9 z 9 z z
so
@ (z; x) 4 1 1
= 1 1 <0= (1; x)
@z 9 z2 z3
therefore
(z; x) < (1; x) = 0 for z > 1 and 0 x 1 (12)
Hence (11) and (12) imply that the bidder with value 0 x 1 bids (x).
In sum, any bidder with any value x has no incentive to deviate from (x), so
(x) is indeed an equilibrium.
Problem 6.5 (Discrete values and signals) Consider a common value …rst-price auc-
tion with two bidders. The common value V can take on only two values, 0 and 1:
Prior to the auction, each bidder i receives a signal Xi which can also take on only
two values, 0 and 1: The joint distribution of the three random variables V , X1 and
X2 is: 8
< 2=9 if x1 = x2 = v
Pr [V = v; X1 = x1 ; X2 = x2 ] = 1=18 if x1 = x2 6= v
:
1=9 if x1 6= x2
Show that the following strategy constitutes a symmetric equilibrium.
a. A bidder with signal Xi = 0 bids E [V j X1 = 0; X2 = 0] = 51 :
b. A bidder with signal Xi = 1 bids randomly over the interval 15 ; 15
8
according
to the distribution
4 5b 1
G (b) =
5 4 5b
(Since the signals and values are discrete, the equilibrium is in mixed strategies.)

33
Solution. The distribution in the question could be illustrated in the table below

V X1 X2 Probability
1 1 1 2/9
1 0 0 1/18
1 1 0 1/9
1 0 1 1/9
0 0 0 2/9
0 1 1 1/18
0 1 0 1/9
0 0 1 1/9

First of all, given bidder 2’s strategy, bidder 1 can not win by bidding below
bidder 2’s lowest bid, 15 , and he does not bid above bidder 2’s highest bid, 15
8
: So bids
1 8
out of 5 ; 15 can not be a pro…table deviation.
If bidder 1 with value X1 = 0 bids b 2 51 ; 15 8
, his expected payo¤ is

1 1 2 1
(b; 0) = (1 b) + (1 b) G (b) + (0 b) + (0 b) G (b)
18 9 9 9
1 1 4 (5b 1) 2 1 4 (5b 1)
= (1 b) + (1 b) b b
18 9 5 (4 5b) 9 9 5 (4 5b)
(5b 1) (9b 12)
=
90 (4 5b)

Because b 2 15 ; 15
8
; the expected payo¤ is negative unless b = 15 , hence bidder 1
with value 0 will not deviate from the strategy and bid 15 :
8
If bidder 1 has value X1 = 1, we need to show that any bid in 15 ; 15 gives him
1 8
the same expected payo¤. Suppose the bidder bid any b 2 5 ; 15 , his expected payo¤
is
2 1 1 1
(b; 1) = (1 b) G (b) + (1 b) + (0 b) G (b) + (0 b)
9 9 18 9
2 4 (5b 1) 1 1 4 (5b 1) 1
= (1 b) + (1 b) b b
9 5 (4 5b) 9 18 5 (4 5b) 9
1
=
15
So bidder with value 1 also does not have any incentive to deviate. So the strategy
constitutes a symmetric equilibrium.

8 Asymmetries and Other Complications


Problem 8.1 (Reserve price) Consider a …rst-price auction with N bidders who
have private values X1 ; : : : ; XN : The vector (X1 ; : : : ; XN ) is distributed over [0; 1]N
according to a density function f which is a¢ liated and symmetric. Suppose that the
seller sets a small reserve price r > 0: Show that the following constitutes a symmetric

34
equilibrium. First, bidders with value x < r do not participate. Second, bidders with
value x r bid Z x
(x; r) = x L (y j x) dy (8.4)
r
where L (y j x) is de…ned in (8.4).

Solution. First, any bidder with value strictly less than r does not participate the
auction because he can not win the object with a price lower than his value.
Second, let us consider a bidder with value x r. He does not bid less than
r because then he has no chance of winning. Suppose that he bids (z) r, his
expected payo¤ is
(z; x) = G (z j x) [x (z)]
Taking the derivative with respect to z yields

@ g (z j x) 0
= G (z j x) (x (z)) (z)
@z G (z j x)

When z = x we have
@ g (z j z) 0
= G (z j z) (z (z)) (z)
@z x=z G (z j z)
= (z (z)) g (z j z) G (z j z) 0 (z)
Z z Z z
g (t j t)
= g (z j z) z z + exp dt dy
r y G (t j t)
2 R z g(tjt) 3
1 exp z G(tjt) dt
G (z j z) 4 R z R z g(tjt) g(zjz)
5
r exp y G(tjt) dt G(zjz) dy
= 0

Similar as in Proposition 6.3, when z < x we have

@ g (z j z) 0
> G (z j x) (z (z)) (z)
@z G (z j z)
@
= =0
@z x=z

where the inequality comes from the fact that

g (z j x) g (z j z)
>
G (z j x) G (z j z)

because of a¢ liation.
@
It can also be veri…ed similarly that @z < 0 when z > x. Thus (z; x) is
maximized by choosing z = x:

35
Problem 8.2 (Increase in number of bidders) Consider a …rst-price auction with N
bidders who have private values X1 ; : : : ; XN : The values X1 ; : : : ; XN are symmet-
rically distributed over [0; 1]N and are a¢ liated. Speci…cally, the joint distribution
1
of values is determined as follows. First, a random variable Z 2 10 ; 2 is drawn
and each value of Z is equally likely. Next, each bidder’s value Xi is drawn from
the distribution FXjZ (x j z) = exp z 1 x 1 over (0; 1]: As in the previous prob-
lem, suppose that the seller sets a reserve price r = 0:5: Let (n) (x; r) denote the
symmetric equilibrium bidding strategy (as in (8.4)) when there are n bidders. Now
suppose that the number of bidders increases to n + 1 and let (n+1) (x; r) denote
the symmetric equilibrium bidding strategy when there are n + 1 bidders (again as in
(8.4)). Show that for some x > r, (n) (x; r) > (n+1) (x; r) ; that is, an increase in
the number of bidders may cause bids to decrease.
(Note: This problem is computationally intensive. A symbolic computation program
will be of great help.)

Solution. The density of joint distribution of X1 ; : : : ; XN conditional on Z = z is

h (x1 ; : : : ; xN j z) = fXjZ (x1 j z) : : : fXjZ (xN j z)

and the joint density of X1 ; : : : ; XN is


1 1
h (x1 ; : : : ; xN ) = h (x1 ; : : : ; xN j z = 0:1) + h (x1 ; : : : ; xN j z = 2)
2 2
1
= f (x1 j 0:1) : : : fXjZ (xN j 0:1)
2 XjZ
1
+ fXjZ (x1 j 2) : : : fXjZ (xN j 2)
2
Conditional on bidder 1’s value X1 = x ; the cumulative distribution function of the
highest bid of the other bidders is

Ry Ry
0 : : : 0 h (x; t2 ; : : : ; tN ) dt2 : : : dtN
G (y j x) = R1 R1
0 : : : 0 h (x; t2 ; : : : ; tN ) dt2 : : : dtN
Ry Ry
0 :R: : 0 12RfXjZ (x j 0:1) fXjZ (t2 j 0:1) : : : fXjZ (tN j 0:1) dt2 : : : dtN
y y
+ 0 : : : 0 12 fXjZ (x j 2) fXjZ (t2 j 2) : : : fXjZ (tN j 2) dt2 : : : dtN
= R1 R1 1 !
: : :
0 R 0 2R XjZ f (x j 0:1) f (t
XjZ 2 j 0:1) : : : f (t
XjZ N j 0:1) dt 2 : : : dtN
1 1
+ 0 : : : 0 12 fXjZ (x j 2) fXjZ (t2 j 2) : : : fXjZ (tN j 2) dt2 : : : dtN
1
Ry Ry
2 fXjZ (x j 0:1) 0R fXjZ (t2 j 0:1) dt2 : : : R0 fXjZ (tN j 0:1) dtN
y y
+ 12 fXjZ (x j 2) 0 fXjZ (t2 j 2) dt2 : : : 0 fXjZ (tN j 2) dtN
= R1 R1 !
1
f (x j 0:1) f (t
0R XjZ 2 j 0:1) dt2 : : : f (t j 0:1) dt
2 XjZ
1 R01 XjZ N N
+ 21 fXjZ (x j 2) 0 fXjZ (t2 j 2) dt2 : : : 0 fXjZ (tN j 2) dtN
1 N 1 N 1
2 fXjZ (x j 0:1) FXjZ (y j 0:1) + 12 fXjZ (x j 2) FXjZ (y j 2)
= 1
2 fXjZ (x j 0:1) + 12 fXjZ (x j 2)

36
and the corresponding density function is
!
1 N 2
2 fXjZ (x j 0:1) (N 1) FXjZ (y j 0:1) fXjZ (y j 0:1)
1 N 2
+ 2 fXjZ (x j 2) (N 1) FXjZ (y j 2) fXjZ (y j 2)
g (y j x) = 1 1
2 fXjZ (x j 0:1) + 2 fXjZ (x j 2)

The expression of G (y j x) and g (y j x) give us the ratio


!
N 2
fXjZ (x j 0:1) (N 1) FXjZ (y j 0:1) fXjZ (y j 0:1)
N 2
g (t j t) +fXjZ (x j 2) (N 1) FXjZ (y j 2) fXjZ (y j 2)
= N 1 N 1
G (t j t) fXjZ (x j 0:1) FXjZ (y j 0:1) + fXjZ (x j 2) FXjZ (y j 2)
2 (N 1 N 2 (N 1 N
0:01t 1) exp 0:1 1 t + 4t 1) exp 2 1 t
=
0:1 [exp (0:1 (1 t 1 ))]N + 2 [exp (2 (1 t 1 ))]N

where the second equality comes from the substitution of the function form of FXjZ .
According to the de…nition in (6:7), we have
Z x
g (t j t)
L (y j x) = exp dt
y G (t j t)
0 ! 1
N
0:01t 2 (N 1) exp 0:1 1 t 1
B Z x N C
B +4t 2 (N 1) exp 2 1 t 1 C
= exp BB dtC
1 N 1 N C
@ y 0:1 (exp (0:1 (1 t ))) + 2 (exp (2 (1 t ))) A

Rx !
y 2 (N 1) t 2 dt
= exp Rx 0:1N t 2 exp 0:1N 1
( ( t 1
))
+ 0:19(N
0:1N
1)
y 0:1 exp(0:1N (1 t 1 ))+2 exp(2N (1 t 1 )) dt
0 1
2 (N 1) x1 y1
= exp @ A
1) R exp(0:1N (1 x ))
1
+ 0:19(N
0:1N exp(0:1N (1 y 1 ))
1
0:1s+2s20
ds
0 1
1 1
2 (N 1) x y
= exp @ exp(0:1N (1 x 1 ))
A
+ 19(N
N
1)
ln s 1
19 ln s19 + 1
20 exp(0:1N (1 y 1 ))
N 1
19 N
1 1 1 1
exp 0:1 (N 1) x y exp 0:1N 1 y + 20
= N 1
1 19 1 N
exp 0:1N 1 x + 20

where the fourth equality comes from changing variable s = exp 0:1N 1 t 1 .

37
Now applying formula (8.4) yields the bidding function with N bidders
Z x
N (x) = x L (y j x) dy
0:5
N 1
19 N
1 1 1 1
Z x exp 0:1 (N 1) x y exp 0:1N 1 y + 20
= x N 1 dy
0:5 1 19 1 N
exp 0:1N 1 x + 20

The following …gure plots the bidding functions with number of bidders N = 2 and
N = 6, where the solid line represents the bidding function with 2 bidders, while the
dash line is the bidding function with 6 bidders. Note that the dash line is below the
solid line for small values, which means for those values the bids are lower with more
bidders.

1.0
bid

0.9

0.8

0.7

0.6

0.5
0.5 0.6 0.7 0.8 0.9 1.0
x
Figure S8.1

Problem 8.3 (A second-price auction without a monotone equilibrium) Consider a


second-price auction with three bidders. Each bidder i receives a private signal Xi .
Bidder 1’s value V1 = X1 +X2 is the sum of his own signal and bidder 2’s signal, while
bidder 2’s and 3’s values are private and equal their own signals— that is, V2 = X2
and V3 = X3 . Each bidder receives either a “high” or a “low” signal. Speci…cally,
X1 2 f1; 2g, X2 2 f0; 4g, and X3 2 f0; 3g. Bidders 1 and 3 have perfectly correlated
signals, both being low with probability 1=2 or both being high with probability 1=2.
Bidder 2’s signal is independent of the others’ signals, being low with probability p
and high with probability 1 p, where p 2 (2=3; 1).
a. Verify that the bidders’ signals are a¢ liated.

38
b. Show that with the speci…ed information structure, there does not exist a pure-
strategy equilibrium of the second-price auction such that the equilibrium strategies
are increasing and undominated.

Solution. Part a.
By the de…nition of a¢ liation in Appendix D, we need to show

Pr x0 _ x00 Pr x0 ^ x00 Pr x0 Pr x00 for all x0 and x00 2 X (13)

If bidder 2’s values are both 0 in x0 and x00 ; (13) becomes


1 2 1 2
p p
4 4
If bidder 2’s values are both 1 in x0 and x00 ; (13) becomes
1 1
(1 p)2 (1 p)2
4 4
If bidder 2’s values are di¤erent in x0 and x00 ; (13) becomes
1 1
(1 p) p (1 p) p
4 4
So the signals are a¢ liated.
Part b. It is easy to see that bidding their values is a undominated strategy
for bidders 2 and 3. Therefore bidder 2 and 3 each employ a pure-strategy which is
increasing. It remains only to compute an undominated strategy for bidder 1.
When X1 = 1, bidder 1 knows that 3 (X3 ) = X3 = 0 . Since 1’s value is
v1 = 1 + X2 , his value is 1 if 2 (X2 ) = X2 = 0 ; while it is 5 is 2 (X2 ) = X2 = 4.
Consequently, 1’s value is always strictly above the high bid of the others and so it
is optimal for him to bid so that he wins with probability one. Because p < 1, this
requires a bid more than 4. However, when X1 = 1 his value is never more than 5,
and so in any undominated equilibrium we must have 1 (1) 2 (4; 5]:
When X1 = 2; bidder 1 knows that 3 (X3 ) = X3 = 3 . Consequently, a bid above
4 still wins with probability one, but bidder 1 now pays 3 when 2 (X2 ) = X2 = 0 and
v1 = 2, and pays 4 when 2 (X2 ) = X2 = 4 and v1 = 6. Hence, a bid above 4 yields
bidder 1 a payo¤ of p (2 3) + (1 p) (6 4), which is negative because p > 2=3.
Similarly, any bid weakly between 3 and 4 yields a negative payo¤. Consequently, we
must have 1 (2) < 3.
Hence, an undominated pure-strategy equilibrium exists, but in no such equilib-
rium is bidder 1’s strategy nondecreasing.

39
9 E¢ ciency and the English Auction
Problem 9.1 (Two-bidder auctions) Suppose that there are two bidders with valua-
tions
v1 (x1 ; x2 ) = 32 x1 + 13 x2
1
v2 (x1 ; x2 ) = 3 x1 + 23 x2
and all signals lie in [0; 1].
a. Using the break-even conditions, …nd an e¢ cient ex post equilibrium of the
English (in this case, also the second-price) auction.
b. Show that the equilibrium strategies so determined survive the iterated elimina-
tion of weakly dominated strategies and are the only strategies to do so.
(Note: Iterated elimination of dominated strategies is carried out stepwise as follows.
In step 1, discard all weakly dominated strategies for both bidders. In step 2, discard
all weakly dominated strategies in the reduced game obtained after step 1. In step
3, discard all weakly dominated strategies in the reduced game obtained after step 2.
Continue in this fashion.)

Solution. Part a. Let 1 (x) and 2 (x) be the prices the bidders drop out at and
1 and 2 be the corresponding inverse functions. According to equation (9.3) in
section 9.2, the break-even conditions are

v1 ( 1 (p) ; 2 (p)) =p

v2 ( 1 (p) ; 2 (p)) =p
which can be rewritten as
2 1
1 (p) + 2 (p) =p
3 3
1 2
1 (p) + 2 (p) =p
3 3
so
1 (p) = 2 (p) =p
Hence the bidding functions are 1 (x) = 2 (x) = x.
Part b. Assume the conditional density functions fX1 jX2 fX2 jX1 are strictly
positive, and 1 ( ) and 1 ( ) are strictly increasing. If bidder 1 with signal x1 bids
b1 , his expected payo¤ is
Z 1
(b1 )
2 2 1
1 (b1 ; x1 ) = x1 + y 2 (y) fX2 jX1 (y j x1 ) dy
0 3 3

and the marginal expected payo¤ is


1
@ 1 2 1 1 fX2 jX1 2 (b1 ) j x1
= x1 + 2 (b1 ) b1 0 1
@b1 3 3 (b1 )

40
1
Step 1. Because x2 2 [0; 1], we have 2 (b1 ) 2 [0; 1] : Therefore if b1 > 32 x1 + 13 ,
the marginal expected payo¤ is
1
@ 1 2 1 1 fX2 jX1 2 (b1 ) j x1
= x1 + 2 (b1 ) b1 0 1
@b1 3 3 (b1 )
1
2 1 fX2 jX1 2 (b1 ) j x1
x1 + b1 0 1
3 3 (b1 )
< 0
where the …rst inequality comes from 2 1 (b1 ) 1. Hence bidder 1 could bene…t by
reducing the bid, so he does not bid more than 23 x1 + 13 : Similarly, if b1 < 23 x1 ; the
marginal expected payo¤ is
1
@ 1 2 1 1 fX2 jX1 2 (b1 ) j x1
= x1 + 2 (b1 ) b1 0 1
@b1 3 3 (b1 )
1
2 1 fX2 jX1 2 (b1 ) j x1
x1 + b1 0 1
3 3 (b1 )
> 0
1
where the …rst inequality comes from 2 (b1 ) 0; hence bidder 1 does not bid less
than 32 x1 : As a result, we have
2 2 1
1 (x1 ) 2 x1 ; x1 +
3 3 3
Applying the same analysis to bidder 2 yields
2 2 1
2 (x2 ) 2 x2 ; x2 +
3 3 3
1
Step 2. From step 1, we have 1 (x1 ) 2 23 x1 ; 23 x1 + 13 , therefore 2 (b1 ) 2
3
2 b1 13 ; 23 b1 : Repeating the same analysis in step 1 implies
4 1 4
1 (x1 ) 2 x1 ; x1
3 3 3
4 1 4
2 (x2 )
x2 2
; x2
3 3 3
Step n: The strategies that survive the n steps of iterated eliminations are

i (xi ) 2[ n xi ; n xi n + 1] if n is odd (14)

i (xi ) 2[ n xi n + 1; n xi ] if n is even (15)


where
2
1 =
3
2
n+1 =
3 1= n

41
2
Taking limit of both sides of the equation above yields lim n = 3 1= lim n
; therefore
n!1 n!1
lim n = 1. Therefore (14) and (15) imply that the set of bidder i’s strategies that
n!1
survives nth iterated elimination shrinks to fxi g as n goes to in…nity. Hence the
equilibrium strategy derived in Part a. is the only strategy that survives the iterated
elimination of weakly dominated strategies.

Problem 9.2 (Ordinality) Suppose is an increasing and di¤ erentiable function


such that (0) = 0. If the valuations v satisfy the average crossing condition, and
for all i, wi ( ) = (vi ( )) then the valuations w also satisfy the average crossing
condition.

Solution. First, note that vi is maximal at x if and only if wi is maximal at x. This


means that for all A N , the set of signals such that the values of all bidders in A
are maximal is the same for both v and w. Then, for any such vector of signals x
which satis…es that for all l 2 A,

vl (x) = max vk (x) vx (16)


k2N

and for all i; j 2 A, i 6= j,

@ wA 1 X @wi
(x) = (x)
@xj ]A i2A @xj
1 X 0 @vi
= (vi (x)) (x)
]A i2A @xj
0 1 X @vi
= (vx ) (x)
]A i2A @xj
0 @
= (vx ) vA (x)
@xj
0 @vi
> (vx ) (x)
@xj
0 @vi
= (vi (x)) (x)
@xj
@wi
= (x)
@xj

where the third equality comes from (16) and the inequality comes from the average
crossing condition for v. Thus, the average crossing condition is also satis…ed by w.

42
10 Mechanism Design with Interdependent Values
All of the problems below concern the following environment. Suppose that there are
two potential buyers for one indivisible object. Each buyer’s private value Xi for
the object is drawn at random from the set X = f10; 20g : Buyers’ values are jointly
distributed as follows:
Pr [10; 10] = Pr [20; 20] = 0:2
Pr [10; 20] = Pr [20; 10] = 0:3
where Pr [x1 ; x2 ] denotes Pr [X1 = x1 ; X2 = x2 ].

Problem 10.1 (Generalized VCG mechanism) What is the generalized VCG mech-
anism (Q ; M ) for the environment speci…ed above? Determine the expected revenue
from this mechanism.

Solution. The buyers’ values are symmetric and private but correlated. Consider
a direct mechanism (QA ; MA ). Recall that Qi (x) is i’s probability of winning the
object when signals x are reported. And Mi (x) includes i’s payment upon “winning”
and “losing” separately. In what follows, these components are written as Miw (x)
and Mil (x); respectively.1
Buyer i’s beliefs about buyer j’s signal conditional on his own are given by Bayes’
rule: i (10 j 10) = i (20 j 20) = 0:4 and i (20 j 10) = i (10 j 20) = 0:6 for both
buyers i = 1; 2. De…ne

X
UiA (zi ; xi ) = i (xj j xi ) QA
i (zi ; xj ) xi
A
Miw (zi ; xj )
xj

+(1 QA
i (zi ; xj )) MilA (zi ; xj ) (17)
to be i’s expected payo¤ when his value is xi , he reports a value of zi and the other
buyer j reports truthfully.
The seller’s expected revenue in the truth-telling equilibrium of this mechanism
(if it exists) is the weighted sum of the payments taken from each buyer i for each
pair of signals x, where weights come from the probability with which each bidder
wins Qi (x) and the probability of the signals P r[x]:
X X
E[RA ] = Pr[x] QA A
i (x)Miw (x) + (1 QA A
i (x))Mil (x) (18)
x i

The generalized Vickrey-Clarke-Groves (VCG) mechanism (Q ; M ) is shown in


the table below.
x Q (x) Miw (x) Mil (x)
1 1
10; 10 2; 2 10 0
10; 20 0; 1 10 0
20; 10 1; 0 10 0
1 1
20; 20 ;
2 2 20 0
1
In Chapter 5, Mi (x) was i’s expected payment and did not depend on who won the object.

43
The buyers have private values so the discrete version of the single crossing condi-
tion (10.4) clearly holds and Proposition 10.1 says that truth-telling is an equilibrium.
The seller’s expected revenue in the truth-telling equilibrium is computed from (18),
above:
1 1 1 1
E[R ] = Pr[10; 10] ( 10 + 0) + ( 10 + 0)
2 2 2 2
+ Pr[10; 20]f(0 10 + 1 0) + (1 10 + 0 0)g
+ Pr[20; 10]f(1 10 + 0 0) + (1 0 + 0 10)g
1 1 1 1
+ Pr[20; 20] ( 20 + 0) + ( 20 + 0)
2 2 2 2
= 0:2 10 + 0:3 10 + 0:3 10 + 0:2 20
= 12

Problem 10.2 Consider the following mechanism. If both buyers report values z1 =
z2 = 20, then pick a buyer randomly with probability 21 , say this is buyer i, and give
him the object for a price of Mi = 20. The other buyer j pays nothing. If both report
values z1 = z2 = 10, again pick a buyer randomly with probability 12 , say i, and give
him the object for a price of Mi = 19: The other buyer, say j, pays Mj = 9 without,
of course, getting the object. If one buyer reports zi = 20 and the other xj = 10, then
give the object to buyer i for a price Mi = 20; The other buyer j receives a transfer
of 6; that is, Mj = 6:
a. Show that the mechanism described above is incentive compatible and individ-
ually rational.
b. What is the expected revenue in the truthful equilibrium of this mechanism?
c. Does the mechanism have other (non-truthful) equilibria?

Solution. Call this mechanism (Q; M). Then

x Q(x) Miw (x) Mil (x)


1 1
10; 10 2; 2 19 9
10; 20 0; 1 20 6
20; 10 1; 0 20 6
1 1
20; 20 2; 2 20 0

Part a. Truth-telling is incentive compatible (IC) for buyer i if for all xi ; zi 2 Xi

Ui (xi ; xi ) Ui (zi ; xi ) (IC)

where Ui was de…ned in (17), above. To verify this condition, it is su¢ cient to consider
only buyer 1 and only z1 6= x1 . That is, we need to show U1 (10; 10) U1 (20; 10) and

44
U1 (20; 20) U1 (10; 20).

1 1
U1 (10; 10) = 0:6f0 (10 20) + 1 6g + 0:4 (10 19) + ( 9)
2 2
=0
1 1
U1 (20; 10) = 0:6 (10 20) + 0 + 0:4f1 (10 20) + 0 6g
2 2
= 7
1 1
U1 (20; 20) = 0:4 (20 20) + 0 + 0:6f1 (20 20) + 0 6g
2 2
=0
1 1
U1 (10; 20) = 0:4 (20 19) + ( 9) + 0:6f0 (20 20) + 1 6g
2 2
=0

So, (IC) is satis…ed. And individual rationality (IR),

Ui (xi ; xi ) Ui (xi ) 0; (IR)

is clearly also satis…ed. Note that this mechanism is e¢ cient and it extracts the entire
surplus.
Part b. The mechanism is IC and IR, so truth-telling is an equilibrium. The
seller’s expected revenue in the truth-telling equilibrium is computed from (18),
above:

E[R] = 0:2(19 + 9) + 0:3(20 6) + 0:3(20 6) + 0:2(20 + 0) = 18

Part c. No, there are no other equilibria. First note that a low-valued buyer will
never report a value of 20 because his payo¤ from doing so is

Pr[opponent reports 10]( 9) + Pr[opponent reports 20]( 5) < 0:

So each buyer must report 10 when his value is 10. Next suppose one buyer reports
10 with probability > 0 when his value is 20. Then the other buyer’s payo¤ when
his value is 10 is
(0:4 + 0:6 )( 9) + 0:6(1 )(6) < 0
which violates individual rationality. So each buyer must report 20 when his value is
20.

Problem 10.3 (Crémer-McLean mechanism) What is the Crémer-McLean mecha-


nism Q ; MC for the environment speci…ed above?

45
Solution. The construction of the Crémer-McLean (CM) mechanism is explained in
the proof of Proposition 10.2. By symmetry of the value structure, we can restrict
attention to buyer 1. The vector of buyer 1’s expected payo¤s in the truth-telling
equilibrium of the VCG mechanism is

U1 (10; 10) 0:4 0 + 0:6 0 0


u1 = =
U1 (20; 20) 0:4 0 + 0:6 10 6

The matrix of buyer 1’s conditional beliefs is

1 (10 j 10) 1 (20 j 10) 0:4 0:6


1 = :
1 (10 j 20) i (20 j 20) 0:6 0:4

This matrix has full row rank, so there is a unique solution to

1 c1 = u1

namely,
c1 (10) 18
c1 = =
c1 (20) 12
The CM mechanism is (Q ; MC ) where

M1C (x1 ; x2 ) = M (x1 ; x2 ) + c1 (x2 ):

Buyer 2’s payments are analagous. The mechanism is summarized in the table below.
The payments made in zero-probability events may be anything; they are written as
in the table.
x Q (x) C (x)
M1w C (x)
M1l C (x)
M2w C (x)
M2l
1 1
10; 10 2; 2 28 18 28 12
10; 20 0; 1 12 28
20; 10 1; 0 28 12
1 1
20; 20 2; 2 8 12 8 12

Problem 10.4 (Non-negativity payo¤ constraint) For the environment speci…ed above,
does there exist a mechanism that (a) is incentive compatible and individually ratio-
nal; (b) gives each buyer a non-negative payo¤ for every realization of the values; and
(c) extracts all the surplus from the buyers?

Solution. No, there is no such mechanism. In the last two mechanisms, the buyers
sometimes made negative payments (that is, the seller paid money to them). Suppose
this is not allowed:
Mik (x) 0 i = 1; 2 k = w; l: (NN)
A revenue-maximizing mechanism will solve the problem

max E[R] subject to (IC), (IR) and (NN): (P)


Q;M

46
This is a linear programming problem. After working it out, you will see that the
best mechanism is a posted price of 20, as given in the table below. The expected
revenue is easily calculated as

Pr[at least one player has a value of 20] 20 = 16:

x Q(x) Miw (x) Mil (x)


10; 10 0; 0 x 0
10; 20 0; 1 20 0
20; 10 1; 0 20 0
20; 20 p; 1 p 20 0

47
11 Bidding Rings
Problem 11.1 (Maximal loss from collusion) Consider a second-price auction with
N 2 bidders. Each bidder’s private value Xi is independently and uniformly dis-
tributed according on [0; 1] :
a. First, suppose bidders bid individually— that is, there is no bidding ring. As
a benchmark, …nd the expected revenue of the seller if he sets an optimal reserve
price r > 0 (as in Chapter 2).
b. Now suppose that the N bidders form a perfectly functioning bidding ring. Find
the expected revenue of the seller if he sets an optimal reserve price r > 0 in
the face of such collusion.
c. Show that for all n; the optimal revenue with collusion is at least one-half of
the optimal revenue without collusion; that is, > 12 .

Solution. Part a. Each player’s expected payment under a reserve price r can be
found using (2.9) and G(y) = F n 1 (y):
Z x
II N
m (x; r) = r + (N 1)y N 1 dy
r
1 N 1 N
= rN + x
N N
The seller’s expected revenue is the sum of the bidders’expected payments:
Z 1
E[R] = N mII (y; r)dy
r
N N 1
= r (1 r) + (1 rN +1 )
(N + 1)
2N N +1 N 1
= r + rN +
N +1 N +1
The seller maximizes this to …nd the optimal reserve price r = 21 . We could also
…nd this by noting that the seller values the object at x0 = 0 and the hazard rate
(x) = 1=(1 x) is increasing in x, so (2.12) is a su¢ cient condition for an optimal
reserve price r . Plugging this into the expression above, we …nd the seller’s expected
revenue under the optimal reserve price:
1N
2 +N 1
=
N +1

Part b. Because the ring consists of all bidders, it does not need to protect itself
from outside bids. If no bidder values the object at least at r, the ring will not bid.
If any ring member has a value at or above r, the ring will submit a bid of r. Hence,
the seller’s expected revenue from setting a reserve price of r is

E[R] = r Pr max xi r = r(1 rN )


i

48
This is maximized at r = (N + 1) 1=N for expected revenue of = N=(N +
1)N +1=N .
Part c. Because r is optimal, the revenue from setting r = 12 instead is less
than or equal to (with equality at N = 1). The …rst line below uses this fact.

1N
1 1 1 1 2 +N 1
1
2 2 2N 2 N +1
N +2
= 2 2(N + 1)
2N
>0

Problem 11.2 (Collusion in …rst-price auctions) Consider a …rst-price auction with


three bidders. Bidder 1’s value X1 = 34 with probability 34 and X1 = 12 with probability
1
4 : Bidders 2 and 3 have …xed and commonly known values. Speci…cally, x2 = 1 and
x3 = 14 :
a. Find an equilibrium of the …rst-price auction when the three bidders act inde-
pendently. (Note: Since values are discrete, this will be in mixed strategies).
b. Now suppose that bidders 1 and 2 form a cartel. While the cartel cannot con-
trol the bids submitted by its members, it can arrange transfers and recommend bids
.Further, suppose that the values of its members become commonly known among the
cartel once it is formed.
i. Find an equilibrium with the cartel, assuming that bidder 3 acts indepen-
dently.
ii. Is it possible for the cartel to ensure that only one member submits a bid?

Solution. Part a. Suppose bidder 2 always bids b2 = 43 and wins, for a payo¤ of
0.25. Bidder 2 is already winning, so bidding more is not better. The strategies of
bidders 1 and 3 must ensure that bidding less is also a bad idea. Let bidder 3 always
bid zero. Suppose that bidding b < 43 gives 2 an expected payo¤ of b=3 < 0:25. Then
bidder 1’s mixed strategy F1 (b) can be solved for b 2 [0; 3=4] by setting

(1 b)F1 (b) = b=3

and so for b 2 [0; 3=4]


b=3
F1 (b) =
1 b
It is straightforward to verify that F1 is indeed a distribution function and that
bidders 1 and 3 cannot do better than follow the strategies given above. There are
many other equilibria leading to the same outcome (allocation of the object and
payments). For example, bidder 1’s strategy may depend on his value.
Part b.i. Suppose the cartel makes no transfers and always lets bidder 2 have
the object. The following argument follows that of part a. Bidder 2 bids b2 = x1 for

49
a payo¤ of 1 x1 ; and bidder 1 uses the mixed strategies F1 (b j x1 ) so that 2 gets
payo¤ b(1 x1 )=x1 < 1 x1 from bidding b < x1 :
1 b
F1 b j x1 = = for b 2 [0; 1=2]
2 1 b
3 b=3
F1 b j x1 = = for b 2 [0; 3=4]
4 1 b
Again, let bidder 3 drop out: b3 = 0. It remains to verify that bidders 1 and 3 are
behaving optimally and that F1 ( j x1 ) is a distribution function for each x1 .
Part b.ii. No, the cartel cannot enforce a one-bidder policy. Suppose bidder 1
has value x1 and does not submit a bid. If the support of bidder 3’s strategy extends
to y > 41 , then bidder 1 will bid b1 < y, trading o¤ a lower payment and a lower
probability of winning. This would make 3’s payo¤ negative, so the support of bidder
3’s strategy must lie inside [0; 14 ]. Bidder 1’s best response is some b1 < 14 . Bidder 2
would like to enter and bid some b2 < 41 for positive payo¤ (as opposed to no payo¤
from staying out). No pre-auction transfers can convince him to stay out. Similarly if
only bidder 2 submitted a bid and that bid was a best response to bidder 3, 1 would
be also choose to enter and bid.
Problem 11.3 (PAKT) A single object is to be sold via a second-price auction to
two bidders whose private values Xi are drawn independently from the uniform distri-
bution on [0; 1]. Suppose that the bidders form a cartel. Find the equilibrium bidding
strategies in the preauction knockout (PAKT).
Solution. First, consider the second-price PAKT. As argued in the book, this is
an incentive-compatible and individually rational direct mechanism. Hence truth-
ful reporting is an equilibrium. Next, suppose the bidders use a …rst-price PAKT.
Equilibrium bidding strategies are given in Proposition 11.3:
1
(x) = x
4

Problem 11.4 (Collusion-proof mechanism) A single object is to be sold to two bid-


ders with private values drawn independently from the uniform distribution on [0; 1].
The following mechanism is used to sell the object. Each bidder i submits a bid bi :
Suppose that bi > bj : Then the loser, bidder j, is asked to pay a …xed amount 13 to the
seller. The winner, bidder i, is awarded the object and asked to pay bi to the losing
bidder j. (If there is a tie, either bidder is assigned the role of a winner.)
a. Find a symmetric equilibrium of this mechanism assuming that the bidders act
noncooperatively.
b. Can the two bidders gain by forming a cartel and colluding against the seller?
Solution. Part a. Suppose is a symmetric equilibrium. If bidder 1 bids (z)
when his value is x, his expected payo¤ in this equilibrium is
Z z Z 1
1
[x (z)] dv + (v) dv:
0 z 3

50
The …rst-order condition evaluated at z = x is
1
x 2 (x) x 0 (x) + = 0:
3
This can be solved using the method of integrating factors. First, rearrange so that
the ’s are on one side:
0 2 1
(x) + (x) = 1 +
x 3x
R
Next, multiply by x2 = exp x2 dxg.

x2 0 (x) + 2x (x) = x2 + x=3


d 2
[x (x)] = x2 + x=3
dx
Integration gives the solution:
1 1
(x) = x +
3 6

Part b. No. Roughly speaking, the bidders cannot take the seller’s piece of
the pie; nor can they make the pie larger. Therefore they have nothing to gain
from collusion. The seller’s pro…t is 31 no matter how the bidders try to collude.
Each bidder’s expected payo¤ in the symmetric equilibrium above is 16 , for a total
expected surplus of 23 . This is also the expectation of the highest value, so no more
surplus is available.

51
13 Equilibrium and E¢ ciency with Private Values
Problem 13.1 (Uniform price auction) Consider a three-unit uniform-price auction
with two bidders. Each bidder’s value vector Xi = X1i ; X2i ; X3i is independently and
identically distributed on the set X = fx 2 [0; 1]3 : x1 x2 x3 g according to a
density function f such that the marginal distributions are:

F1 (x1 ) = (x1 )2
F2 (x2 ) = (2 x2 ) x2

F3 is left unspeci…ed. Show that the bidding strategy (x1 ; x2 ; x3 ) = (x1 ; (x2 )2 ; 0)
constitutes a symmetric equilibrium of the uniform-price auction.

Solution. Suppose one bidder with values (x1 ; x2 ; x3 ) bids (b1 ; b2 ; b3 ) where 1 b1
b2 b3 0: Let (y1 ; y2 ; y3 ) be the values for the other bidder. The expected payo¤
of the bidder is
Z
= [x1 + x2 + x3 3y1 ] dF (y1 ; y2 )
fy1 <b3 g
Z h i
+ x1 + x2 2 max (y2 )2 ; b3 dF (y1 ; y2 )
fb2 >(y2 )2 ;b3 <y1 g
Z
+ [x1 b2 ] dF (y1 ; y2 )
fb1 >0;b2 <(y2 )2 g

which is equivalent to
Z
= [x1 + x2 + x3 3y1 ] dF (y1 ; y2 )
fy1 <b3 g
Z
+ [x1 + x2 2b3 ] dF (y1 ; y2 )
f(y2 )2 <b3 <y1 g
Z h i
+ x1 + x2 2 (y2 )2 dF (y1 ; y2 )
fb3 <(y2 )2 <b2 g
Z
+ [x1 b2 ] dF (y1 ; y2 )
fb2 <(y2 )2 g

and then to
Z
= x1 b2 + [x3 3y1 + 2b3 ] dF1 (y1 )
fy1 <b3 g
Z h i
+ x2 2 (y2 )2 + b2 dF2 (y2 )
f(y2 )2 <b2 g
Z h i
+ 2 (y2 )2 2b3 dF2 (y2 )
f(y2 )2 <b3 g

52
Finally,
Z b3
= x1 b2 + (x3 3y1 + 2b3 ) 2y1 dy1
0
Z p
b2
+ x2 2 (y2 )2 + b2 (2 2y2 ) dy2
0
Z p
b3
+ 2 (y2 )2 2b3 (2 2y2 ) dy2
0
1p p 8 32
= x1 b2 + b23 x3 + b2 2b2 + 6x2 3 b2 x2 + b23 b
3 3 3
where the second equality comes from b1 > 0 and y1 y22 . So the marginal payo¤s
are
@
=0
@b1
@ 1
2
1
= x2 b2 1 1 b22
@b2

@ 1
= 2x3 b3 + 2b3 4b32
@b3
1 1
= b32 (2x3 + 2) b32 4

It is easy to check that every marginal payo¤ is zero when (b1 ; b2 ; b3 ) = x1 ; (x2 )2 ; 0 :
Moreover, @
@b2 0 if b2 < (x2 )2 and @
@b2 0 if b2 > (x2 )2 ; and @
@b3 0 if b3 > 0,
2
hence (x1 ; x2 ; x3 ) = x1 ; (x2 ) ; 0 is indeed an equilibrium.

Problem 13.2 (Uncertain supply) Consider a multiunit uniform-price auction with


N bidders each of whom has use for one unit only. At the time of bidding, the actual
number of units that will be available for sale is uncertain and could range anywhere
between 1 and K where K < N: Show that it is a weakly dominant strategy for each
bidder to bid his or her value.

Solution. The expected payo¤ for one bidder with value v bids b is
K
X
Pr (k) (b; v; k)
k=1

where Pr (k) is the probability of k unit supply and (b; v ; k) is his expected payo¤
conditional on k units of supply. In section 13.4, we already know it is weakly
dominant strategy for him to bid his value without supply uncertainty, so

(v; v; k) (b; v; k) for k = 1; : : : ; K

53
so
K
X K
X
Pr (k) (v; v; k) Pr (k) (b; v; k)
k=1 k=1

which means it is also weakly dominant strategy to bid his value with uncertain
supply.

Problem 13.3 (Multiple equilibria) Consider a two-unit uniform-price auction with


two bidders. Each bidder’s value vector Xi is identically and independently distributed
so that the marginal distributions of the values of both goods is uniform, that is,
F1 (x1 ) = x1 and F2 (x2 ) = x2 : Show that for any increasing function (z) such that
0 (z) z; the bidding strategy (x1 ; x2 ) = (x1 ; (x1 )) constitutes a symmetric
equilibrium.

Solution. Suppose bidder 1 has value (x1 ; x2 ) and the other bidder’s strategy is
(x1 ; x2 ) = (x1 ; (x1 )). We already know that he bids x1 for the …rst unit in Section
13.4, so suppose he deviates to (x1 ; b) ; then his expected payo¤ is:
Z b Z 1 (b) Z 1 (x
1)
= 2(x1 y)dy + (x1 b)dy + (x1 (y))dy
0 b 1 (b)

The marginal expected payo¤ is

@ 1 1
= 2(x1 b) + 0( 1 (b))
1 (x1 b) (b) b
@b
1 1
0( 1 (b))
(x1 ( (b)))
1
= x1 (b)

It is easy to check that when b = (x1 ) ; @@b = 0. Moreover, if b > (x1 ), @@b < 0 and
if b < (x1 ), @@b > 0. Hence (x1 ; x2 ) = (x1 ; (x1 )) is indeed an equilibrium.

54
15 Sequential Sales
Problem 15.1 (Power distribution) Consider a situation in which two identical ob-
jects are to be sold to three interested bidders in two auctions conducted sequentially.
Each bidder has use for at most one item— there is single-unit demand. Bidders’pri-
vate values are identically and independently distributed according to the distribution
F (x) = x2 on [0; 1] :
a. Find a symmetric equilibrium bidding strategy if a sequential …rst-price format
is used.
b. Find a symmetric equilibrium bidding strategy if a sequential second-price for-
mat is used.
c. Compare the distribution of prices in the two auctions under the …rst- and
second-price formats.

Solution. Part a. Using (15.4) with F (x) = x2 , we …nd


Z x
I 1
2 (x) = ydF (y)
F (x) 0
Z x
1
= 2 2y 2 dy
x 0
2
= x:
3
Using (15.6), we …nd
Z x
I 1
1 (x) = 2 (y)dF 2 (y)
F (x) 0 2
Z x
1 8 4
= 4 y dy
x 0 3
8
= x:
15
Part b. By Proposition 15.3,

II 2
1 (x) = x
3
II
2 (x) = x:

Part c. Renumber the bidders so that X1 > X2 > X3 . The distribution of prices
from the …rst-price sequential auction is given by

LI (p1 ; p2 ) = Pr maxf I
1 (X1 );
I
1 (X2 );
I
1 (X3 )g p1 ; maxf I
2 (X2 );
I
2 (X3 )g p2
8 8 2 8 2
= Pr X1 p1 ; X2 p1 ; X 2 p2 ; X3 p1 ; X 3 p2
15 15 3 15 3
2
15 15 3
=F p1 F min p1 ; p2 :
8 8 2

55
And the price distribution for the second-price sequential auction is
2
3 3
LII (p1 ; p2 ) = F p1 F min p1 ; p 2 :
2 2

It is clear that LI LII so prices in the second-price sequential auction stochastically


dominate those from the …rst-price.

Problem 15.2 (Multiunit demand) Consider a situation in which two identical ob-
jects are to be sold to two interested bidders in two second-price auctions conducted
sequentially. Bidders have multiunit demand with values determined as follows. Each
bidder draws two values Z1 and Z2 from the uniform distribution F (z) = z on [0; 1] :
The bidder’s value for the …rst unit is X1 = max fZ1 ; Z2 g and his marginal value for
the second unit is X2 = min fZ1 ; Z2 g : (This is just an instance the multiuse model
discussed in Chapter 13.)
a. Show that the following strategy constitutes a symmetric equilibrium of the
sequential second-price format:
i. in the …rst auction, bid 1 xi1 ; xi2 = 21 xi1 ; and
ii. in the second auction, bid truthfully— that is, bidder i bids xi2 if he won
the …rst auction; otherwise he bids xi1 .
b. Show that the sequence of equilibrium prices (P1 ; P2 ) is a submartingale— that
is, E [P2 j P1 = p1 ] p1 and with positive probability, the inequality is strict.

Solution. Part a. We will show that 1 (x1 ; x2 ) = 21 x1 is the unique strictly


increasing strategy that only depends on the …rst-period value. Because the …rst-
period strategy in the proposed equilibrium depends only on the …rst-period value,
we can de…ne ( ) 1 ( ; y) for any y. Suppose bidder 1 has values x = (x1 ; x2 ),
and bidder 2 is following the equilibrium strategy. In the second round, it is optimal
for bidder 1 to also bid truthfully. In the …rst round, he won’t bid more than his
opponent’s highest possible bid (1) or less than his lowest (0). So any bid b he
makes with positive probability (his strategy may be random or dependent on x2 ),
must satisfy b = (t) for some t 2 [ (0); (1)]. If he bids as type t x1 , his
expected payo¤ is
Z t Z x2
+ 2
(t; x) = (x1 (y1 ))d[F (y1 )] + (x2 y1 )d[F 2 (y1 )]
0 0
Z x1
+ 2(1 F (t)) (x1 y2 )d[F (y2 )]:
0

The …rst line shows his expected payo¤ if the …rst round is won; and the second,
his expected payo¤ from losing the …rst round. As long as bidder 2 has not won
an object, he faces both of his opponent’s value draws. The higher of these has
distribution F 2 (y1 ). If his opponent has won one unit already, he only faces one draw
with distrubution F . Bidder 1 can lose to either of bidder 2’s draws, so the last
integral is multiplied by 2. Now suppose he bids as type t 2 (x2 ; x1 ). His expected

56
payo¤ is
Z t Z x2
2
(t; x) = (x1
(y1 ))d[F (y1 )] + (x2 y1 )d[F 2 (y1 )]
0 0
Z x1
+ 2(1 F (x1 )) (x1 y2 )d[F (y2 )]
0
Z x1 Z y 1
+2 (x1 y2 )d[F (y2 )] d[F (y1 )]:
t 0

On the …rst line is his expected payo¤ from winning the …rst round; on the second,
from losing to an opposing bid y1 > x1 ; and on the third, from losing to y1 x1 . In
the last integral, it is imposed that y1 > y2 , so each has distribution F . To show that
behaving as type t = x1 is optimal, …rst we will show that it satis…es the necessary
…rst-order conditions. After that, we must show that the payo¤ from bidding any
other t gives weakly lower payo¤. The …rst-order conditions are

@ + (t; x) @ (t; x)
0 and 0:
@t @t
t=x1 t=x1

Written out, these are


Z x1
2f (x1 )F (x1 )(x1 (x1 )) 2f (x1 ) (x1 y2 )f (y2 )dy2 0 and
Z0 x1
2f (x1 )F (x1 )(x1 (x1 )) 2f (x1 ) (x1 y2 )f (y2 )dy2 0
0

Putting these together and re-arranging gives a formula for :


R x1
0 (x1 y1 )f (y1 )dy1
(x1 ) = x1 (19)
F (x1 )

Plugging in F (x) = x and f (x) = 1 gives (x1 ) = 12 x1 , as desired. This demonstrates


that satis…es the necessary …rst-order condition. However, we need to show that
bidding according to is optimal when the other player is playing . So we must
show that (x1 ) gives weakly higher payo¤ than bidding as type t x2 . The payo¤
from t x2 is
Z t Z t
2
(t; x) = (x1 (y1 ))d[F (y1 )] + (x2 y1 )d[F 2 (y1 )]
0 0
Z x1
+ 2(1 F (x1 )) (x1 y2 )d[F (y2 )]
0
Z x1 Z y 1
+2 (x1 y2 )d[F (y2 )] d[F (y1 )];
t 0

where each line has the same interpretation as for , the only di¤erence being that
now, upon winning in the …rst round, he knows y1 t. Taking the derivative and

57
substituting in and F , it is easily seen that @ =@t is increasing on its domain
[0; x2 ]. Also, (x2 ) = + (x1 ), so it is weakly optimal to follow strategy .
Part b. Let X1 X2 be the values of the bidder who wins the …rst round and
Y1 Y2 the other bidder’s values. There are three equilikely orderings of these values,
so we can express the desired expectation in terms of them:
1
E[P2 j P1 = p1 ] = E[P2 j X1 > X2 > Y1 > Y2 ; P1 = p1 ]
3
1
+ E[P2 j X1 > Y1 > X2 > Y2 ; P1 = p1 ]
3
1
E[P2 j X1 > Y1 > Y2 > X2 ; P1 = p1 ]
3
In the …rst case P1 = (Y1 ) = 12 Y1 and P2 = Y1 . Because is strictly increasing,
knowing P1 = p1 means knowing that the price tomorrow will be ; 1 (p1 ) = 2p1 .
For the second case, the prices are (Y1 ) and X2 . X2 is the higher of two random
draws from the uniform distribution on [0; 2p1 ], so its expectation is 32 2p1 . In the
…nal case, the prices are also (Y1 ) and X2 . This time X2 is the lower of two random
draws from the uniform distribution on [0; 2p1 ], so its expectation is 13 2p1 . Plugging
these into the equation above, we have
1 2 1 4
E[P2 j P1 = p1 ] = 2p1 1 + + = p1 > p1 :
3 3 3 3

Problem 15.3 (Multiunit demand) Consider the same environment as in the previ-
ous problem.
a. Show that the following strategy also constitutes a symmetric equilibrium of
the second-price format:
i. in the …rst auction, bid 1 xi1 ; xi2 = xi2 ; and
ii. in the second auction, bid truthfully.
b. What can you say about the resulting sequence of equilibrium prices (P1 ; P2 )?

Solution. Part a. The following argument is similar to that found in Problem 15.2.
We will show that 1 (x1 ; x2 ) = x2 is the unique strictly increasing strategy that only
depends on the second-period value. If the second player is bidding according to a
strictly increasing function (y2 ) in the …rst round and truthfully in the second,
then it is still optimal for bidder 1 to bid truthfully in the second round. Bidder 1’s
payo¤ from bidding as type t 2 (x2 ; x1 ) in the …rst round is
Z tZ 1
+
(t; x) = 2 (x1 (y2 ))dF (y1 )dF (y2 )
0 y2
Z x2 Z x2
+2 (x2 y1 )dF (y1 )dF (y2 )
0 y2
Z x1 Z1
+2 (x1 y2 )dF (y1 )dF (y2 )
t y2

58
The …rst two lines are the payo¤ from winning the …rst round; and the last is the
payo¤ from losing it. The payo¤ from bidding t < x2 is
Z tZ 1
(t; x) = 2 (x1 (y2 ))dF (y1 )dF (y2 )
0 y2
Z tZ x2
+2 (x2 y1 )dF (y1 )dF (y2 )
0 y2
Z x1 Z 1
+2 (x1 y2 )dF (y1 )dF (y2 )
t y2

The …rst two lines are the payo¤ from winning the …rst round; and the last is the
payo¤ from losing it. The …rst-order conditions lead to a characterization of :
Z 1 Z 1
(x1 (x2 ))dF (y1 ) (x1 x2 )dF (y1 ) = 0:
x2 x2

This is clearly satis…ed by (x2 ) = x2 . We still need to show that the payo¤ from
bidding t x1 is no greater than the payo¤ from this strategy. After writing out the
payo¤ ++ from such a strategy and showing it is decreasing in t, we could show
that the payo¤ at t = x1 is the same as at t = x2 , so it is weakly optimal to bid x2 .
Part b. Consider the three possible orderings identi…ed in Problem 15.2.a, above:
if X1 > X2 > Y1 > Y2 , the price sequence is Y2 < Y1 ; if X1 > Y1 > Y2 > X2 , the
price sequence is X2 < Y2 ; and if X1 > Y1 > X2 > Y2 , the price sequence is Y2 < X1 .
So the price sequence in this equilibrium is increasing.

59
16 Nonidential Objects
Problem 16.1 (Low revenue) Consider the problem of allocating a set of two objects
in K = fa; bg to three buyers with values as follows:

a b ab
x1 0 0 10 + "
x2 10 10 10
x2 10 10 10

where 0 " < 1:


a. Find an e¢ cient allocation and the corresponding payments in the VCG mech-
anism.
b. What is the total revenue accruing to the seller?

Solution. Part a. It would be e¢ cient to give a to buyer 2 and b to buyer 3. With


buyer 2, the other two buyers have total welfare of W 2 (x) = 10. Without buyer 2
(or equivalently if buyer 2 reported x2 = 0), it would be e¢ cient to give both items
to buyer 1 for a total welfare of W 2 (0; x 2 ) = 10 + ". So 2’s payment in the VCG
mechanism is W 2 (0; x 2 ) W 2 (x) = ". Similarly, buyer 3 would pay ".
Part b. The seller collects 2" in revenue.

Problem 16.2 (Complements) Consider the problem of allocating a set of four ob-
jects in K = fa1 ; a2 ; b1 ; b2 g to …ve buyers. Buyer 1 has use only for objects a1 and b1 ;
buyers 2 and 3 have use only for objects a2 and b2 ; buyer 4 has use only for b1 and
b2 ; and buyer 5 has use only for objects a1 and a2 . Speci…cally, the values attached
by the buyers to these bundles are

x1 (a1 b1 ) = 10
x2 (a2 b2 ) = 20
x3 (a2 b2 ) = 25
x4 (b1 b2 ) = 10
x5 (a1 a2 ) = 10

All other combinations (or packages) are valued at zero.


a. Find an e¢ cient allocation and the corresponding payments in the VCG mech-
anism.

Solution. It is e¢ cient to give a1 b1 to buyer 1 and a2 b2 to buyer 3. With buyer 1,


the total welfare of the other buyers is W 1 (x) = 25. If buyer 1 reported a value
vector of zeros, it would be e¢ cient to give a2 b2 to buyer 3 and a1 and b1 to no one,
for total welfare of W 1 (0; x 1 ) = 25. Hence buyer 1 pays 0 in the VCG mechanism.
With buyer 3 present, the other buyers have total welfare W 3 (x 3 ) = 10. If buyer
3 were not present, it would be e¢ cient to give a1 b1 to buyer 1 and a2 b2 to buyer 2
for total welfare W 3 (0; x 3 ) = 30. So buyer 3 pays 20.

60
Problem 16.3 Show that the conditions (16.6) and (16.7) are equivalent.

Solution. (16:6) ) (16:7). Suppose (16.6) holds and take two sets S; T K.
Because K is …nite, we can list T n S = a1 ; :::; an . De…ne T 0 T; S 0 T \ S and
T m = T m 1 [ fam g and S m = S m 1 [ fam g for m = 1; :::; n. Applying (16.6) n
times to S m T m 63 am+1 gives the following sequence of inequalities:

x(S 1 ) X(S 0 ) x(T 1 ) x(T 0 )


x(S 2 ) X(S 1 ) x(T 2 ) x(T 1 )
:::
n 1 n 2
x(S ) X(S ) x(T n 1
) x(T n 2
)
n n 1 n n 1
x(S ) X(S ) x(T ) x(T )

Adding these inequalities up gives

x(S n ) x(S 0 ) x(T n ) x(T 0 )

And substituting S n = (T \ S) [ (T n S) = S, S 0 = T \ S, T n = T [ (T n S) = T [ S
and T 0 = T gives
x(S) x(T \ S) x(T [ S) x(T )
which can be rearranged to make (16.7).
(16:7) ) (16:6). Suppose (16.7) holds and take a 2 K; a 2
= T; and S T . De…ne
S 0 = S [ fAg. Applying (16.7) to the sets S 0 ; T gives

x(S [ fag) + x(T ) x(T [ fag) + x(S)

which can be rearranged as (16.6).

61
17 Packages and Positions
Problem 17.1 (Ine¢ ciency without package bidding) Suppose that there are two
objects, a and b, for sale and two bidders with the following values

a b ab
x1 y z 2
x2 2 2 2

where y and z are parameters that lie between 0 and 1: Argue that an ascending
auction format in which bidders can only bid on a and b individually, and not on the
package ab; cannot allocate e¢ ciently. (Note: Without package bidding, the price of
the package ab is necessarily the sum of the prices of the individual objects a and b.)

Solution. Consider the ascending auction format described in Section 17.1 with
increments ". Without loss of generality, let 0 < z y < 1 so that an e¢ cient
allocation gives a to bidder 1 and b to bidder 2 (or also the reverse when y = z).
Suppose that there is some ascending auction that leads to this outcome and payments
(pa ; pb ). So that bidder 2 does not bid on a we need pb pa . So that bidder 1 does
not attempt to win both goods, we need pa + pb 2. Together, these inequalities
imply that pa = pb = 1. Moreover, y = 1 since 1’s payo¤ y pa must be nonnegative.
This contradicts y < 1 so for any y and z strictly between zero and one, no ascending
auction without package bidding can lead to an e¢ cient outcome.

Problem 17.2 (Gross Substitutes) Show that if bidder i with value vector xi satis…es
the gross substitutes condition (de…ned on page 241) then xi satis…es the substitutes
condition (de…ned in (16.6)). Equivalently, show that the gross substitutes condition
implies that xi (S) is submodular.

Solution. This argument is taken from the proof of Lemma 5 in Gul and Stacchetti
(1999).2 Take A 2 K, T 63 A and S T and let M > x(K) be a price at which goods
are too expensive. De…ne a price vector
(
0 a 2 T [ fAg
pa = :
M a 62 T [ fAg

Recall that values are monotone: if A B then x(A) x(B). As a result, T [ fAg 2
D(p). For each " 0, de…ne a price vector
8
>
<" a=A
T
qa (") = 0 a2T :
>
:
M a 62 T [ fAg
2
Gul, F. and E. Stacchetti (1999), “Walrasian Equilibrium with Gross Substitutes,” Journal
of Economic Theory, 87, 95–124.

62
De…ne a critical price for good A by

" = maxf" : T [ fAg 2 D(qT ("))g:

By construction we have
T [ fAg 2 D(qT (" )): (20)
Also for " > " we have qT (") p and T [ fAg 2 D(p) so by gross substitutes there
must be a set C 2 D(qT (")) with T C. Goods outside T [ fAg are too expensive
so we must also have C T [ fAg. However, by the de…nition of " we know that
T [ fAg 62 D(qT (")). Hence T [ fAg 2 D(qT (")). This holds for all " > " , so by
upper semi-continuity of D it also holds for " = " (see Gul and Stachetti for details):

T 2 D(qT (" )): (21)

Together (20) and (21) give

x(T [ fAg) " = x(T ): (22)

De…ne a price vector 8


>
<" a=A
S
qa = 0 a2S :
>
:
M a 62 S [ fAg
We have qS qT (" ) and T [ fAg 2 D(qT (" )), so by gross substitutes there must
be a set C 2 D(qS ) containing S [ fAg = fa 2 T [ fAg : qaS = qaT (" )g. On the
other hand, all other goods are una¤ordable so C S [ fAg and S [ fAg 2 D(qS ).
Because of this latter fact, we have

x(S [ fAg) " x(S):

Combining this with (22) gives the substitutes condition (16.6):

x(S [ fAg) x(S) " = x(T [ fAg) x(T ):

Problem 17.3 (Complements) Suppose that the objects in K = fa1 ; a2 ; b1 ; b2 g are


sold via the ascending proxy auction. There are …ve interested bidders Bidder 1 has
use only for objects a1 and b1 ; bidders 2 and 3 have use only for objects a2 and b2 ;
bidder 4 has use only for b1 and b2 ; and bidder 5 has use only for objects a1 and a2 .
Speci…cally, the values attached by the bidders to these bundles are

x1 (a1 b1 ) = 10
x2 (a2 b2 ) = 20
x3 (a2 b2 ) = 25
x4 (b1 b2 ) = 10
x5 (a1 a2 ) = 10

63
All other combinations (or packages) are valued at zero. (The speci…cation is the
same as in Problem 16.2.)
a. Show that the gross substitutes condition is violated.
b. Show that it is not an equilibrium for each bidder to report truthfully to his
proxy.

Solution. Part a. Suppose p(a1 ) = 10; p(b1 ) = 0; p0 (a1 ) = 10; p0 (b1 ) = 1. Then
p0 p, D1 (p) = fa1 b2 g and D1 (p0 ) = ;. Let S = a1 b1 2 D1 (p). The set fa : a 2
S; p(a) = p0 (a)g = a1 is not contained in any S 0 2 D1 (p0 ) so bidder 1 violates the
gross substitutes condition.
Part b. If all the other bidders are reporting truthfully, bidder 3 would do better
by reporting x3 (a2 b2 ) = 20 + ". Hence truthful reporting is not an equilibrium.

Problem 17.4 (Identical objects) Consider a situation in which K identical units are
to be allocated among N identical buyers. Each buyer has the same K-dimsional value
vector x; where x (k) denotes the total value from obtaining k units. Suppose that the
units have decreasing marginal values— that is, x (k + 2) x (k + 1) x (k + 1)
x (k) : An allocation is then simply a vector of the form s = s1 ; s2 ; :::; sN where si
is the number P of units awarded to buyer i: For any I N; de…ne the surplus function
w (I) = maxs Ii=1 x(si ): Show that w is submodular, that is, for any I < J < N

w (I + 1) w (I) w (J + 1) w (J)

(Note: This is just a specialization of Lemma 17.1 to the case of identical objects and
identical buyers.)

Solution. It is su¢ cient to prove that w (I + 1) w (I) w (I + 2) w (I + 1) :


When there are I winners, the welfare is maximized by allocation r1 ; : : : ; rI where
the ith winner has ri units. Similarly, (s1 ; : : : ; sI+1 ) is the allocation maximizing the
welfare when there are I + 1 winners, and t1 ; : : : ; tI+2 is the allocation maximizing
the welfare when there are I + 2 winners. Because the objects are identical, we
can rearrange the order of winners so that ri+1 ri for i = 1; : : : I, si+1 si for
i = 1; : : : I + 1 and ti+1 ti for i = 1; : : : I + 2.
First, applying x(k + 2) x (k + 1) x(k + 1) x (k) multiple times, we have

x (s + 1) x (s) x s0 + 1 x s0 if s0 s: (23)

which means losing one unit when there are more units is better than losing one unit
when there are less units.
Second, because of decreasing marginal value, the objects are allocated as evenly
as possible when the welfare is maximized, therefore the di¤erence in si is at most
one, so
sI+1 s1 + 1: (24)
and the ith winner has less units when there are more winners

ri si ti for i = 1; : : : ; I; and sI+1 tI+1 (25)

64
Third, we have

s s0 x s0 + 1 x s0 x (s) x s0 (26)
0 0
s s [x (s) x (s 1)] if s s: (27)

because

x (s) x s0 = [x (s) x (s 1)] + [x (s 1) x (s 2)] +


0 0
::: + x s + 1 x s
[x (s) x (s 1)] + [x (s) x (s 1)] +
: : : + [x (s) x (s 1)]
0
= s s [x (s) x (s 1)]

where the inequality comes from (23). Roughtly, (27) means the losing s s0 units
is better than s s0 times the welfare loss by decreasing one units.
Then we have
I+1
X I
X
w (I + 1) w (I) = x si x ri
i=1 i=1
I
X
= x sI+1 x ri x si
i=1

Thus,
I
X
w (I + 1) w (I) x sI+1 ri si x si + 1 x si
i=1
XI
x sI+1 ri si x s1 + 1 x s1
i=1
I+1 I+1
= x s s x s1 + 1 x s1
x sI+1 sI+1 x sI+1 x sI+1 1
I+2 I+1 I+2 I+1
= x t +x s x t s x sI+1 x sI+1 1

65
But the right-hand side of the inequality is no less than

x tI+2 + sI+1 tI+2 x sI+1 x sI+1 1


I+1 I+1 I+1
s x s x s 1
I+2 I+2 I+1
= x t +t x s x sI+1 1
I+1
X
= x tI+2 ti si x sI+1 x sI+1 1
i=1
I+1
X
x tI+2 ti si x si x si 1
i=1
I+1
X
x tI+2 x si x ti
i=1
= w (I + 2) + w (I + 1)

where the …rst, fourth and sixth inequalities come from (??) and (27), the second and
…fth equalities come from (23), and the third inequality comes from (24) and (27).
Roughly speaking, the intuition is as follows. The second row is the welfare
increase by changing from allocation (r1 ; : : : ; rI ) to (s1 ; : : : ; sI+1 ); and the third row
break this welfare increase into two parts, the welfare increase from the new winner
I + 1 and the welfare loss from the old winners 1; : : : ; I: The …rst inequality means for
each winner’s, losing ri si units is better than the losing ri + 1th unit ri si times,
and the second inequality means losing one unit when there are si (more) units is
better than losing one unit when there are s1 (less) units, and the other inequalities
can be interpreted similarly.

Problem 17.5 (GSP) Three positions are to be assigned among four bidders. The
positions have “click” values of 1 > 2 > 3 : Each of the four bidders have “per-
click” values of x1 > x2 > x3 > x4 : These values are commonly known and so we are
considering a situation with complete information.
a. Find an equilibrium of the sealed-bid generalized second-price auction (GSP).
b. Is the equilibrium unique? If not, characterize all equilibria of the GSP.

Solution. Part a. Consider the following strategies:

b1 = x1
1 1
b2 = M (x)= 1 = [( 1 2 )x
2
+( 2 3 )x
3
+ 4
3x ]
1
3 2 1 3 4
b = M (x)= 2 = [( 2 3 )x + 3x ]
2
3 1
b4 = M (x)= 3 = 3x
4
= x4
3

66
Each bidder makes the same payment as in the VCG mechanism. Bidder 1 will not
deviate to slot 2 for value v 1 (2) because

v 1 (1) = 1x
1
[( 1 2 )x
2
+( 2 3 )x
3
+ 4
3x ]
1 3 4
2x [( 2 3 )x + 3 x ]:

Similarly, none of the other bidders wish to win di¤erent slots at the associated prices.
Part b. Here we follow Varian (2007) closely.
Let I = f1; 2; 3; 4g be the set of bidders and K = f1; 2; 3; 4g the set of slots, with
a rate of 4 = 0 clicks for the fourth slot. An outcome of the auction is a mapping
: K ! I. An outcome is supported by some equilibrium bids if the following hold:
(k)
8k 2 K k (x b (k+1) ) 0
(k)
8k; ` 2 K; ` < k k (x b (k+1) ) ` (x
(k)
b (`+1) )
(k)
8k; ` 2 K; ` > k k (x b (k+1) ) ` (x
(k)
b (`) )
b (1) b (2) b (3) b (4) b5 0 (28)

The …rst line is the individual rationality condition; the next two lines are incentive
compatibility conditions; and the last must hold for good k to be assigned to bidder
(k) by the auction. These inequalities de…ne the set of bids that can support an
outcome . Some outcomes may only be supported for certain parameter values x; .
With some abuse of notation, we will refer to such an equilibrium by the allocation
alone.
It is clear that the equilibrium is not unique. Suppose is an equilibrium outcome
supported by bids b . Then bidder (1) can bid !b (1) for any ! > 1 and it is still an
equilibrium. A subset of “symmetric”equilibria may be characterized as follows (see
Varian, 2007): for all k; ` 2 K
(k) (`)
k (x pk ) ` (x p` ) (29)

or equivalently
x (k) ( k `) k pk ` p` ;

where pk bk+1 for k = 1; 2; 3; 4. To show that this condition characterizes a subset


of equilibria, we must show that it implies the equilibrium conditions above.
Claim. Condition (29) implies the conditions (28) and that is e¢ cient (that is,
it is the identity mapping).
Proof of claim. To show the …rst line of the equilibrium conditions, note that
4 = 0 so
(k) (4)
k (x pk ) 4 (x p4 ) = 0:
The second line is a special case of (29). To show that is the identity mapping, it
is su¢ cient to show that x (k) x (k+1) . For any k; ` the condition gives

x (k) ( k `) k pk ` p`

x (`) ( ` k) ` p` k pk :

67
Adding these gives
(x (k) x (`) )( k `) 0
so (k) (`) if and only if k `. So the slots are assigned e¢ ciently and we can
write k instead of (k). To show that fourth line holds (decreasing prices), note that
k k
k (x pk ) k 1 (x pk 1)

can be rearranged for

pk 1 k 1 pk k + xk ( k 1 k)
pk k + pk ( k 1 k)
= pk k 1:

The second inequality uses the …rst line of the equilibrium conditions: xk pk .
Finally, to show that the third line holds:
k k k
k (x pk ) ` (x p` ) ` (x p` 1 ):

This completes the proof.


Second claim. If the condition (29) holds between k and k 1 and between k and
k + 1 for all k, then it holds for all pairs.
Sketch of proof. Suppose it holds between 1 and 3 and between 2 and 3. We will
only show that it also holds between 2 and 3. Adding

x1 ( 1 2) p1 1 p2 2

and
x2 ( 2 3) p2 2 p3 3 ) x1 ( 2 3) p2 2 p3 3

gives
x1 ( 1 3) p1 1 p3 3

which is the condition for k = 1; ` = 3. For the other direction, add

x3 ( 3 2) p3 3 p2 2 , x3 ( 2 3) p2 2 p3 3
2 3
x ( 2 1) p2 2 p1 1 ) x ( 1 2) p1 1 p2 2

to get
x3 ( 1 3) p1 1 p3 3 , x3 ( 3 1) p3 3 p1 1

which completes the sketch. Finally, we can use this claim to characterize prices in
symmetric equilibria. The conditions between k and k + 1 are
k k k+1 k+1
k (x pk ) k+1 (x pk+1 ) and k+1 (x pk+1 ) k (x pk )
xk ( k k+1 ) + pk+1 k+1 pk k xk+1 ( k k+1 ) + pk+1 k+1
k k+1
x (1 k) + k pk+1 pk x (1 k ) + k pk+1

68
where each of the lines above is equivalent and k k+1 = k. So, the set of symmetric
equilibrium bids is given by the following inequalities:

x3 b4 x4
x2 (1 3) + b4 3 b3 x3 (1 3) + b4 3
1 3 2 2 3
x (1 2) +b 2 b x (1 2) +b 2
1 1 2
b x (1 1) +b 1

This completes the characterization of symmetric equilibria of the GSP auction.


Random-outcome equilibria. There are also equilibria with random outcomes, ei-
ther because some winning bids are tied or because players are randomizing. Suppose
each bidder i bids independently according to a random variable B i with no mass
points. Then i’s payo¤ to bidding b is
3
X
v i (b; B i ) = P rfb Yk g k [x
i
E(Yk j b > Yk )]:
k=1

where Yk is the k th highest order statistic of the other bidders’bids. These strategies
form a Nash equilibrium if, for all i 2 I, all b; b0 2 suppB i and all b00 62 suppB i we
have

v i (b; B i ) = v i (b0 ; B i )
and v i (b; B i ) v i (b00 ; B i ):

69

You might also like